Sie sind auf Seite 1von 175

1304 211 Chemical Engineering Principles

and Calculations

1304 211 Chemical


Engineering Principles
and Calculations

By
Assist. Prof. Dr. Wipada Sanongraj

Chapter I : Introduction to
Chemical Engineering
Calculations

By Assist. Prof. Dr. Wipada Sanongraj 1


1304 211 Chemical Engineering Principles
and Calculations

What do chemical engineers


do?
Petroleum Government
Biotechnology Lime and cement
Consulting Man-made fibers
Drugs and Metallurgical and metal
products
pharmaceuticals
Paints, varnishes and
Fats and oils pigments
Fertilizer and agricultural Pesticides and herbicides
chemical Plastic materials and
Foods and beverages synthetic resins

1.1 Units and dimensions


Objectives of this section
Add, subtract, multiply, and divide units
associated with numbers
Specify the basic and derived units in the SI and
American Engineering system
Convert one set of units in equation into another
equivalent set for mass, length, area, etc..
Define and know how to use the gravitational
conversion factor, gc

By Assist. Prof. Dr. Wipada Sanongraj 2


1304 211 Chemical Engineering Principles
and Calculations

1.1 Units and dimensions


Dimensions are basic concepts of
measurement
Such as length, time, mass, temperature

Units are the means of expressing the


dimensions
Such as cm, hour, kg

Rules of operation
Only same units can be added or subtracted
For example, 10 lbs + 5 grams
5 kgs + 3 Joules
1 ft + 3 sec
1 horsepower + 30 watts

Different units can be multiplied or divided

By Assist. Prof. Dr. Wipada Sanongraj 3


1304 211 Chemical Engineering Principles
and Calculations

Example 1.1: Dimensions and


Units
Add the following
A) 1 ft + 3 s
B) 1 horsepower + 300 watts
A) has no meaning since the dimensions
of the two terms are not the same
B) 1 hp = 746 watts,
746 watts + 300 watts = 1046 watts

Example of SI units
Physical Quantity Name of Unit Symbol Definition of Unit
Basic SI Units
Length meter m
Mass kilogram kg
Temperature kelvin K
Time second s
Amount of substance mole mol
Derived SI Units
Energy Joule J Kg.m2.s-2
Force Newton N Kg.m.s-2 J.m-1
Power watt W Kg.m2.s-3 J.s-1
Density Kilogram per cubic meter Kg.m-3
Acceleration Meter per second squared m.s-2
Velocity Meter per second m.s-1
Pressure Newton per square N.m-2, Pa
meter, pascal
Heat Capacity Joule per J.kg-1.K-1
(kilogram.kelvin)
Alternative Units
Time minute, hour, day, year min, h, d, y
o
Temperature Degree Celsius C
Volume liter L
Mass ton, gram t, g

By Assist. Prof. Dr. Wipada Sanongraj 4


1304 211 Chemical Engineering Principles
and Calculations

Example of American
Engineering System Units

Physical Quantity Name of Unit Symbol


Basic Units
Length feet ft
Mass pound (mass) lbm
Force pound (force) lbf
Time second, hour s, hr
o
Temperature degree Rankine R
Derived Units
Energy British thermal unit, foot pound Btu, ft.lbf
(force)
Power horsepower hp
Density pound(mass) per cubic foot lbm/ft3
Velocity feet per second ft/s
Acceleration feet per second squared ft/s2
Pressure Pound(force) per square inch lbf/in2

SI Prefixes

Factor Prefix Symbo Factor Prefix Symbo


l l
109 giga G 10-1 deci d
106 mega M 10-2 centi c
103 kilo k 10-3 milli m
102 hecto h 10-6 micro m
101 deka da 10-9 nano n

By Assist. Prof. Dr. Wipada Sanongraj 5


1304 211 Chemical Engineering Principles
and Calculations

Example 1.2: Conversion of


Units
If a plane travels at twice the speed of
sound (assume that the speed of sound
is 1100 ft/s), how fast is it going in
miles per hour
Solution

2 1100 ft 1 mi 60 s 60 min
s 5280 ft 1 min 1 hr

= 1500 mi/hr (mph)

Example 1.3: Use of Units

Change 400 in3/day to cm3/min


Solution

400 in.3 2.543 cm3 1 day 1 hr


day 1 in3. 24 hr 60 min

= 4.56 cm3/min

By Assist. Prof. Dr. Wipada Sanongraj 6


1304 211 Chemical Engineering Principles
and Calculations

At the sea level at 45 °latitude, the numerical value


of the average acceleration of gravity (g) is 32.174
ft/s2.
The acceleration of gravity varies by a few tenths
of 1% from place to place on the surface of the
earth
The inverse of the conversion factor with the
numerical value 32.174 is given the special symbol,
gc
gc = 32.174 (ft)(lbm)/(s2)(lbf)
From gc/g = 1, a one pound mass is to weight one
pound
Weight can be defined as the opposite of the force
required to support a mass
The pound mass and pound force are not the same
units in American Engineering system.

Example 1.4: Use of gc

One hundred pounds of water is flowing through a


pipe at the rate of 10.0 ft/s. What is kinetic energy
of this water in (ft)(lbf)?

Solution

K = 1 100 lbm (10 ft)2 1


2 s2 32.174 (ft)(lbm)/(s2)(lbf)

= 155 (ft)(lbf)

By Assist. Prof. Dr. Wipada Sanongraj 7


1304 211 Chemical Engineering Principles
and Calculations

Example 1.5: Use of gc


What is the potential energy in (ft)(lbf) of a 100 lb drum
hanging 10 ft above the surface of the earth with
reference to the surface of the earth?
Solution
Assume that the 100 lb means 100 lb mass
g = acceleration of gravity = 32.2 ft/s2
P = 100 lbm 32.2 ft 1 10 ft
s2 32.174(ft)(lbm)/(s2)(lbf)
= 1001 (ft)(lbf)

Dimensional Consistency
The equations must be dimensionally consistent in
order to be able to operate.
Each term must have the same net dimensions and
units in the equation.
For example, van der Waals equation
atm.cm6
cm3
a
(P + )(V - b ) = RT
V2
atm cm3

By Assist. Prof. Dr. Wipada Sanongraj 8


1304 211 Chemical Engineering Principles
and Calculations

1.2 The mole unit


In the SI a mole is composed of 6.02×1023
molecules
In the American engineering system, a
pound mole is composed of
6.02×1023×453.6 molecules
To convert the number of moles to mass,
we use molecular weight
What is the unit of MW?

1.2 The mole unit


(continue)

the g mol = (mass in g)/(molecular weight)

the lb mol = (mass in lb)/(molecular weight)


Or
mass in g = (mol. Wt)(g mol)
mass in lb = (mol. Wt)(lb mol)

By Assist. Prof. Dr. Wipada Sanongraj 9


1304 211 Chemical Engineering Principles
and Calculations

Example 1.6: Use of MW


If a bucket holds 2.00 lb of NaOH (mol. Wt. = 40),
how many
a) Pound moles of NaOH does it contain?
b) Gram moles of NaOH does it contain?
Solution
(a) 2.00 lb NaOH 1 lb mole NaOH = 0.05 lb mol
40.0 lb NaOH

= 22.7 g mol
(b) 2.00 lb NaOH 1 lb mol NaOH 454 g mol
40 lb NaOH 1 lb mol

Example 1.7: Use of MW

How many pounds of NaOH are in 7.5 g mol


of NaOH?
Solution
Basis: 7.50 g mol of NaOH
7.50 g mol NaOH 1 lb mol 40 lb NaOH
454 g mol 1 lb mol NaOH
= 0.661 lb NaOH

By Assist. Prof. Dr. Wipada Sanongraj 10


1304 211 Chemical Engineering Principles
and Calculations

1.3 Conventions in Methods of


Analysis and Measurement
1.3.1 Density
Density is the ratio of mass per unit volume
Unit is kg/m3 or lb/ft3
Density of water is 1 g/cm3 or 62.4 lb/ft3
Density of liquid and solid do not change
significantly with changing of pressure but
change with temperature

Density

Liquid density Density of a mixture of ethyl alcohol


And water as a function of composition

By Assist. Prof. Dr. Wipada Sanongraj 11


1304 211 Chemical Engineering Principles
and Calculations

1.3.2 Specific Gravity


It is the ratio of two densities, density of the
interested substance and density of a
reference substance
Thought of as dimensionless unit
For liquids and solids, a reference substance
is usually water
For gas, a reference substance is air
To be precise, state the temperature at
which each density is chosen

Example 1.8: Density and


SPGR
If dibromopentane (DBP) has a specific
gravity of 1.57, what is the density in (a)
g/cm3 (b) lbm/ft3 and (c) kg/m3
Solution
(a) 1.57 g DBP 1.0 g H2O
= 1.57 g DBP
cm3 cm3 cm3
1.0 g H2O
cm3

(b) 1.57 g DBP 106 cm3 1 m3 2.20*10-3 lb = 97.97 lbm DBP


ft3
cm3 1 m3 35.31 ft3 1g

By Assist. Prof. Dr. Wipada Sanongraj 12


1304 211 Chemical Engineering Principles
and Calculations

Or 1.57 g/cm3 DBP 62.4 lb/ft3 62.4 lb/ft3


= 97.97 lbm DBP
1 g/cm3 ft3
1 g/cm3 H2O 62.4 lb/ft3
1 g/cm3

(c) 1.57 g DBP (100 cm)3 1 kg


cm3 1 m3 1000 g = 1.57 × 103 kg DBP
m3

Or

1.57*103 kg DBP 1.0×103 kg H2O


m3 m3
1.0*10 kg H2O
3

m3

Be Cautious!!!!!

For the mixture system, an average specific


gravity can not be determined by
multiplying the individual component
specific gravities or densities by respective
mass fractions of the components and
summing the products.

By Assist. Prof. Dr. Wipada Sanongraj 13


1304 211 Chemical Engineering Principles
and Calculations

Example 1.9: Application of


SPGR

In the production of a drug having a


molecular weight of 192, the exit stream
from the reactor flows at the rate of 10.3
L/min. The drug concentration is 41.2% (in
water), and the specific gravity of the
solution is 1.025. Calculate the
concentration of the drug (in kg/L) in the
exit stream, and the flow rate of the drug in
kg mol/min.

Solution
First, we need to transform the mass
fraction of 0.412 into mass per liter of the
drug. Take 1.00 kg of the exit solution as a
basis
Basis: 1.00 kg solution

0.412 kg Drug
Reactor
0.588 kg Water

By Assist. Prof. Dr. Wipada Sanongraj 14


1304 211 Chemical Engineering Principles
and Calculations

Density of solution = 1.025 g soln 1.0 g H2O


cm3 cm3
1.0 g H2O
cm3
= 1.025 g soln/cm3
Next
0.412 kg drug 1.025 g soln 1 kg soln 103 cm3
1.00 kg soln 1 cm3 103 g soln 1L

= 0.422 kg drug/L soln

To get the flow rate, we take a different basis, 1


minute

Basis: 1 min = 10.3 L solution

10.3 L soln 0.422 kg drug 1 kg mol drug


1 min 1 L soln 192 kg drug

= 0.0226 kg mol/min

By Assist. Prof. Dr. Wipada Sanongraj 15


1304 211 Chemical Engineering Principles
and Calculations

1.3.3 Mole Fraction and Mass (Weight) Fraction

Mole fraction is the ratio of moles of particular


substance and the total moles
For example, if the mixture has 2 components, A and B.
The composition of A is 2 moles and B is 4 moles.
xA = moles of A = 2 = 0.334
total moles 2+4
xB = moles of B = 4 = 0.667
total moles 2+ 4

XA + XB = 0.334 + 0.667 = 1.00

Mole fraction of A = moles of A


total moles

Mass fraction of A = mass of A


total mass

By Assist. Prof. Dr. Wipada Sanongraj 16


1304 211 Chemical Engineering Principles
and Calculations

Example 1.10: Mole fraction


and mass fraction
An industrial-strength drain cleaner contains
5.0 kg of water and 5 kg of NaOH. What are
the mass fraction and mole fraction of each
component in the drain cleaner container?
Solution
Basis: 10 kg of total solution

Component kg Weight Mol. Wt. kg mol Mole


fraction fraction
H2O 5.0 5.0/10 18.0 0.278 0.278/0.
403
=0.69

NaOH 5.0 5.0/10 40.0 0.125 0.125/0.


403
=0.31

Total 10.0 1.0 0.403 1.0

By Assist. Prof. Dr. Wipada Sanongraj 17


1304 211 Chemical Engineering Principles
and Calculations

1.3.4 Concentrations
Concentration is the quantity of some solute
per specified amount of solvent, or solution,
in a mixture of two or more components
(1) mass per unit volume (lbm of solute/ft3, g of
solute/L, lbm of solute/bbl, kg of solute/m3)
(2) mole per unit volume (lb mol of solute/ft3, g
mol of solute/L, g mol of solute/cm3)
(3) Parts per million (ppm), parts per billion
(ppb)
(4) Other methods of expressing concentration
e.g., molarity (g mol/L) and normality
(equivalents/L)

Example 1.11: Use of ppm


The current OSHA 8 hour limit for HCN in air is 10.0
ppm. A lethal dose of HCN in air (from the Merck
index) is 300 mg/kg of air at room temperature.
How many mg HCN/kg air is the 10.0 ppm? What
fraction of the lethal dose is 10.0 ppm?
Solution
Basis: 1 kg mol of the air/HCN mixture
(a) 10 ppm = 10 g mol HCN = 10 g mol HCN
10 air + HCN gmol 106 g mol air
6

10 g mol HCN 27.03 g HCN 1 g mol air 103 mg HCN 103 g air
106 g mol air 1 g mol HCN 29 g air 1 g HCN 1 kg air

= 9.32 mg HCN/ kg air


(b) 9.32/300 = 0.031

By Assist. Prof. Dr. Wipada Sanongraj 18


1304 211 Chemical Engineering Principles
and Calculations

1.4 Basis
To chose the basis, ask yourself these
questions
What do I have to start with?
What answer is called for?
What is the most convenient basis to use?

Example 1.12: Choosing


Basis
The dehydrogenation of the lower alkanes has
been carried out using a ceric oxide catalyst.
What is the mass fraction and mole fraction of
Ce and O in the catalyst?
Solution
No answers for the 1st and 2nd questions, so a
convenient basis would be to take 1 kg mol
because we know the mole ratio of Ce to O in
the compound.

By Assist. Prof. Dr. Wipada Sanongraj 19


1304 211 Chemical Engineering Principles
and Calculations

Basis: 1 kg mol CeO

Component kg mol Mole Mol. kg Mass


fraction Wt Fraction
Ce 1 0.5 140 140 0.9

O 1 0.5 16 16 0.1

Total 2 1.0 156 1.0

Example 1.13: Changing


Basis

A medium-grade bituminous coal analyzes as follows


Component %
S 2
N 1
O 6
Ash 11
Water 3

The residuum is C and H in the mole ratio H/C = 9.


Calculate the weight fraction of the coal with the ash
and moisture omitted.

By Assist. Prof. Dr. Wipada Sanongraj 20


1304 211 Chemical Engineering Principles
and Calculations

Solution
Basis: 100 kg Coal
The sum of S+N+O+ash+water is 2+1+6+11+3=23
kg
Hence the C and H must be 100-23=77 kg
To determine the kilograms of C and H, we have to
select a new basis. Because C/H is molar ratio
Basis: 100 kg mol of C and H
Component Mole kg mol Mol. Wt kg
Fraction
H 9/10 0.9 90 1.008 90.7

C 1/10 0.1 10 12 120

Total 1.0 100 210.7

Finally, return to the original basis, we have

H: 77 kg 90.7 kg H
210.7 kg total = 33.15 kg H

C: 77 kg120 kg C
210.7 kg total = 43.85 kg C
Summarized table
Component kg Wt. Fraction
C 43.85 0.51
H 33.15 0.39
S 2 0.02
N 1 0.01
O 6 0.07
Total 86.0 1.00

By Assist. Prof. Dr. Wipada Sanongraj 21


1304 211 Chemical Engineering Principles
and Calculations

1.5 Temperature
The temperature is a measure of thermal
state considered in reference surrounding
Units of temperature: Fahrenheit (F),
Celsius (C), Rankine (R), Kelvin (K)
Absolute temperature scales have their
zero point at the lowest possible
temperature that can exist.

Temperature measuring
instruments span the range
from near absolute zero to
beyond 3000 K.

By Assist. Prof. Dr. Wipada Sanongraj 22


1304 211 Chemical Engineering Principles
and Calculations

Temperature scales

Conversion between scales


TR = TF + 460
TK = TC + 273
DF = DR
DC = DK
(DC/DF) = 1.8
(DK/DR) = 1.8
C/5 = (F-32)/9
F-32 = C*1.8

By Assist. Prof. Dr. Wipada Sanongraj 23


1304 211 Chemical Engineering Principles
and Calculations

Example 1.14: Temperature


Conversion
Convert 100 °C to (a) K, (b) ° F, and (c) ° R
Solution
(a) 100 °C + 273 = 373 K
(b) 100 °C /5*9 + 32 = 212 ° F
(c) 212 ° F + 460 = 672 ° R
Or
(373 K)(1.8 oR/1 K) = 672 oR

Example 1.15: Temperature


Conversion
The thermal conductivity of aluminum at 37
°F is 117 Btu/(hr)(ft2)(°F/ft). Find the
equivalent value at 0oC in terms of
Btu/(hr)(ft2)(K/ft)
Solution
117 (Btu)(ft) 1.8 D°F 1 D°C
(hr)(ft2)(°F) 1 D°C 1 DK

= 211 Btu/(hr)(ft2)(K/ft)

By Assist. Prof. Dr. Wipada Sanongraj 24


1304 211 Chemical Engineering Principles
and Calculations

Conversion
The heat capacity of sulfuric acid in a handbook has
the units J/(g mol)(°C) and is given by the relation
heat capacity = 139.1 + 1.56 ×10-1T
Where T is expressed in °C. Modify the formula so
that the resulting expression has the associated units
of Btu/(lb mol)(°R) and T is in ° R

Heat capacity = {139.1+1.56×10-1[(T°R-460-32)/1.8]}*


1 J 1 Btu 454 g mol 1 °C
(g mol)(°C) 1055J 1 lb mol 1.8oR

Heat capacity = 23.06+ 2.07 ×10-2T °R

1.6 Pressure
Pressure is normal force
per unit area Pressure at the bottom of the
static column of water is
p = F/A = rgh + p0
p = pressure at the bottom of the
column of the fluid
F = force
A = area
r = density of fluid
g = acceleration of gravity
h = height of the fluid column
p0 = pressure at the top of the
column of fluid

By Assist. Prof. Dr. Wipada Sanongraj 25


1304 211 Chemical Engineering Principles
and Calculations

Example 1.17: Pressure


Suppose the cylinder of fluid is a column of mercury that has an
area of 1 cm2 and is 50 cm high. Density of mercury is 13.55
g/cm3. Calculate the force exerted by the mercury alone on the
1 cm2 section of the bottom plate.
Solution
F = 13.55 g 980 cm 50 cm 1 cm2 1kg 1m 1N
cm 3 s2 1000 g 100cm (kg)(m)/s2
= 6.64 N
The pressure on the section of the plate covered by the mercury is
the force per area of the mercury plus the pressure of
atmosphere
P = 6.64 N (100 cm)2 (1 m2)(1 Pa) 1 kPa
1 cm2 1 m2 1N 1000 Pa
= 66.4 kPa + p0

Pressure can be expressed by either absolute or


relative scales.
An open-end manometer would measure a
relative pressure (gauge pressure).
Closing off the end of the manometer would
measure an absolute pressure.
Atmospheric pressure measured by barometer is a
barometric pressure
Gauge pressure + barometric pressure = absolute
pressure
Units of pressure: mm Hg, ft H2O, atm, bar, psi,
kgf/cm2, Pascal

By Assist. Prof. Dr. Wipada Sanongraj 26


1304 211 Chemical Engineering Principles
and Calculations

Open-end manometer
Barometer

Absolute pressure manometer

“C” Bourdon Spiral Bourdon

Visual Bourdon gauge reads zero


pressure when open to the atmosphere.
Pressure sensing device in the Bourdon
gauge is a thin metal tube

By Assist. Prof. Dr. Wipada Sanongraj 27


1304 211 Chemical Engineering Principles
and Calculations

Standard atmosphere is the pressure


equivalent to 1 atm or 760 mm Hg at 0
oC

Atmospheric pressure is a variable and


is measured by a barometer.

Standard atmosphere:
760 mm Hg, 29.92 in Hg
33.91 ft H2O
1 atm
1.013 bars
14.7 psia
1.013*105 Pa or N/m2 or 101.3 kPa

By Assist. Prof. Dr. Wipada Sanongraj 28


1304 211 Chemical Engineering Principles
and Calculations

Example 1.18: Pressure


Conversion
The pressure gauge on a tank of CO2 used
to fill soda-water bottles reads 51.0 psi.
At the same time the barometer reads
28.0 in Hg. What is the absolute
pressure in the tank in psia?

Solution
The pressure gauge is reading psig
Absolute pressure = gauge pressure +
atmospheric pressure
Basis: barometric pressure = 28 in Hg
Atmospheric pressure = 28 in Hg 14.7 psia
29.92 in Hg
= 13.76 psia
Absolute pressure in the tank
51.0 + 13.76 = 64.8 psia

By Assist. Prof. Dr. Wipada Sanongraj 29


1304 211 Chemical Engineering Principles
and Calculations

Example 1.19: Pressure


Conversion
Air is flowing through a duct under a
draft of 4.0 cm H2O. The barometer
indicates that the atmospheric pressure
is 730 mm Hg. What is the absolute
pressure of the gas in inch Hg?

Solution
We can ignore the gas density above the
manometer fluid.
Basis: 730 mm Hg
Atmospheric pressure = 730 mm Hg 29.92 in Hg
760 mm Hg
= 28.7 in Hg
Basis: 4 cm H2O draft (under atmospheric)
4 cm H2O 1 in 1 ft 29.92 in Hg = 0.12 inHg
2.54 cm 12 in 33.91 ft H2O
The reading is 4 cm H2O draft, the absolute reading
in uniform units is
28.7-0.12 = 28.6 in Hg

By Assist. Prof. Dr. Wipada Sanongraj 30


1304 211 Chemical Engineering Principles
and Calculations

Example 1.20:Vacuum
Pressure
Small animals can live at reduced air
pressure down to 20 kPa. In a test a
mercury manometer attached to a tank
reads 64.5 cm Hg and the barometer
reads 100 kPa. Will the mice survive?

Solution
Basis: 64.5 cm Hg below atmospheric
the absolute pressure in the tank is
100 kPa – 64.5 cm Hg 101.3 kPa
76 cm Hg
= 100 – 86 = 14 kPa absolute
The mice will not survive

By Assist. Prof. Dr. Wipada Sanongraj 31


1304 211 Chemical Engineering Principles
and Calculations

Example 1.21: pressure


differences
In measuring the flow of fluids in a
pipeline, a differential manometer can
be used to determine the pressure
difference across the orifice plate. The
flow rate can be calibrated with the
observed pressure drop. Calculate the
pressure drop p1-p2 in pascal for the
manometer in Figure.

Solution
p1-p2 = (rf-r)gd
= (1.1-1)103 kg 9.807 m 22*10-3 m 1 Ns2 1 Pa*m2
m3 s2 kg*m 1 N

= 21.6 Pa

By Assist. Prof. Dr. Wipada Sanongraj 32


1304 211 Chemical Engineering Principles
and Calculations

1.7 The Chemical Equation


and Stoichiometry
C7H16 + 11O2 7CO2 + 8 H2O
Stoichiometric ratios (coefficients)
1 mole of heptane will react with 11 moles
of oxygen to give 7 moles of carbon
dioxide and 8 moles of water
These maybe lb mole, g mole, kg mole.

Example 1.22: Chemical


Equation
if 10 kg of C7H16 react completely with the
stoichiometric quantity of O2, how much kg
of CO2 will be found as products? On the
basis of 10 kg.

1 kg mol of C7H16 will react with O2 to form 7 kg


mol of CO2
10 kg C7H16 1 kg mol C7H16 7 kg mol CO2 44 kg CO2
100 kg C7H16 1 kg molC7H6 1 kg mol CO2

= 30.8 kg CO2

By Assist. Prof. Dr. Wipada Sanongraj 33


1304 211 Chemical Engineering Principles
and Calculations

Example1.23: Chemical
Equation
In the combustion of heptane, CO2 is
produced. Assume that you want to
produce 500 kg of dry ice per hour and that
50% of the CO2 can be converted into dry
ice, as shown in Figure. How many kg of
heptane must be burned per hour
Other products CO2 gas 50%

CO2 solid dry ice 50%


C7H16 gas
Reactor
500 kg/hr

Solution
Basis: 500 kg of dry ice (equivalent to 1 hr)
C7H16 + 11 O2 7 CO2 + 8 H2O

500 kg dry ice 1 kg CO2 1 kg mol CO2 1 kg mol C7H16 100 kg C7H16
0.5 kg dry ice 44 kg CO2 7 kg mol CO2 1 kg mol C7H16

= 325 kg C7H16

By Assist. Prof. Dr. Wipada Sanongraj 34


1304 211 Chemical Engineering Principles
and Calculations

Example 1.24:
Stoichiometry
A limestone analysis
CaCO3 92.89%
MgCO3 5.41%
Insoluble 1.70%
(a) How many pounds of calcium oxide can
be made from 5 tons of this limestone?
(b) How many pounds of CO2 can be
recovered per pound of limestone?
(c) How many pounds of limestone are
needed to make 1 ton of lime?

Solution
Basis: 100 lb of limestone
CO2

Limestone

CaO
Heat MgO Lime
Insoluble
CaCO3 CaO + CO2

MgCO3 MgO + CO2

Substance CaCO3 MgCO3 CaO MgO CO2

Mol. Wt. 100.1 84.32 56.08 40.32 44.0

By Assist. Prof. Dr. Wipada Sanongraj 35


1304 211 Chemical Engineering Principles
and Calculations

Limestone Products
Component lb = % lb mol Solid lb CO2 (lb)
Component
CaCO3 92.89 0.9280 CaO 52.04 40.83

MgCO3 5.41 0.0642 MgO 2.59 2.82

Insoluble 1.70 Insoluble 1.70

Total 100.00 0.9920 Total 56.33 43.65

92.89 lb CaCO3 1 lb mol CaCO3 1 lb mol CaO 56.08 lb CaO


= 52.04 lb
100.1 lb CaCO3 1 lb mol CaCO3 1 lb mol CaO

5.41 lb MgCO3 1 lb mol MgCO3 1 lb mol MgO 40.32 lb MgO


= 2.59 lb
84.32 lb MgCO3 1 lb mol MgCO3 1 lb mol MgO

(a) CaO produced = 52.04 lb CaO 2000 lb 5 ton


= 5200 lb CaO
100 lb stone 1 ton

(b) CO2 recovered = 43.65 lb CO2 = 0.437 lb

100 lb stone

(c) Limestone required = 100 lb stone 2000 lb = 3550 lb stone

56.33 lb lime 1 ton

By Assist. Prof. Dr. Wipada Sanongraj 36


1304 211 Chemical Engineering Principles
and Calculations

Things to realize !!
In industrial reactors you will rarely find exact
stoichiometric amounts of materials used.
Limiting reactant is the reactant that is present in
the smallest stoichiometric amount.
For example
A + 3B + 2C = P
1.1 mol of A, 3.2 mol of B, and 2.4 mol of C are fed
as reactants in the reactor, we choose A as the
reference substance and calculate
Ratio in feed Ratio in chemical equation
B/A 3.2/1.1 = 2.91 3/1 = 3
C/A 2.4/1.1 = 2.18 2/1 = 2

B is the limiting reactant relative to A,


and A is the limiting reactant relative to
C, hence B is the limiting reactant
B<A<C

By Assist. Prof. Dr. Wipada Sanongraj 37


1304 211 Chemical Engineering Principles
and Calculations

Excess reactant is a reactant present in


excess of the limiting reactant. The
percent excess of a reactant is based on
the amount of any excess reactant above
the amount required to react with the
limiting reactant

% excess = moles in excess/moles


required to react with the limiting
reactant *100

For example: C7H16 + 11O2 = 7CO2 + 8H2O

If we have 12 mol of O2 and 1 mol of C7H16


Ratio in feed Ratio in chemical equation
O2/C7H16 12/1 = 12 11/1 = 11

% excess O2 = (12-11)/11*100 = 9.1%


In this example, C7H16 is the limiting reactant

By Assist. Prof. Dr. Wipada Sanongraj 38


1304 211 Chemical Engineering Principles
and Calculations

Conversion is the fraction of the feed or


some key material in the feed that is
converted into products

%conversion = 100* (moles or mass of


feed that react)/(moles or mass of feed
introduced)

For example; if there is 10 kg of C7H16 and 14.4 kg of CO2 are


formed in the reaction of C7H16,
% conversion of C7H16 = mol of C7H16 reacts/mol of C7H16 in
feed
Mol of C7H16 reacts =
14.4 kg CO2 1 kg mol CO2 1 kg mol C7H16
44 kg CO2 7 kg mol CO2 = 0.0468 kg mol
Mol of C7H16 in feed =
10 kg C7H16 1 kg mol
100 kg C7H16 = 0.1 kg mol C7H16
%conversion = 0.0468/0.1*100 = 46.8%

By Assist. Prof. Dr. Wipada Sanongraj 39


1304 211 Chemical Engineering Principles
and Calculations

Selectivity is the ratio of the moles of a


particular product produced to the moles
of another product produced
For example; methanol can be converted
into ethylene or propylene by the
reactions
2CH3OH = C2H4 + 2 H2O
3CH3OH = C3H6 + 3 H2O
Selectivity of C2H4 relative to C3H6 at 80%
conversion of CH3OH = 0.19/0.08 = 2.4
mol C2H4/molC3H6

Products from the conversion of methanol

By Assist. Prof. Dr. Wipada Sanongraj 40


1304 211 Chemical Engineering Principles
and Calculations

Yield is a weight or moles of final


product divided by the weight or moles
of initial or key reactant either fed or
consumed

Example 1.25: Incomplete


Reaction
From this reaction, suppose that 0.6 kg of
stibnite and 0.25 kg of iron are heated
together to give 0.2 kg of SB metal determine
A) the limiting reactant
B) the percentage of excess reactant
C) the degree of completion
D) the percent conversion
E) the yield

Sb2S3 + 3Fe = 2Sb + 3FeS

By Assist. Prof. Dr. Wipada Sanongraj 41


1304 211 Chemical Engineering Principles
and Calculations

Solution
Component kg MW g mol
Sb2S3 0.6 339.7 1.77
Fe 0.25 55.85 4.48
Sb 0.2 121.8 1.64
FeS 87.91

1.77 g mol Sb2S3


FeS
4.48 g mol Fe Reactor

1.64 g mol Sb

A) to find the limiting reactant, examine ratio of Sb2S3 to


Fe, 1/3 = 0.33. In the actual reaction the ratio is 1.77/4.48
= 0.4, hence Sb2S3 is the excess reactant and Fe is the
limiting reactant. Sb2S3 required to react with the Fe is
4.48/3 = 1.49 g mol

B) The percentage of excess reactant is


% excess = (1.77-1.49)/1.49*100 = 18.8 excess Sb2S3

C) Calculate how much Fe react from 1.64 Sb formed


1.64 g mol Sb 3 g mol Fe
2 g mol Sb = 2.46 g mol Fe
Degree of completion = 2.46/4.48 = 0.55

By Assist. Prof. Dr. Wipada Sanongraj 42


1304 211 Chemical Engineering Principles
and Calculations

D) percent conversion of Sb2S3


1.64 g mol Sb 1 g mol Sb2S3
2 g mol Sb = 0.82 g mol Sb2S3
% conversion = 0.82/1.77*100 = 46.3%

E) The yield of Sb formed per kg of Sb2S3 fed to


reaction
Yield = 0.2 kg Sb/0.6 kg Sb2S3
= 0.33 kg Sb/1kg Sb2S3

Chapter 2
Material Balance

By Assist. Prof. Dr. Wipada Sanongraj 43


1304 211 Chemical Engineering Principles
and Calculations

The objectives in studying this chapter are

* Define a system and draw the system


boundaries for which the material balance is to
be made.

* Explain the different between an open and


closed system.

* Write the general material balance in words


including all terms. Be able to apply the balance
to simple problems.

System : any arbitrary portion or whole of a


process set out specifically for analysis.

System boundary : circumscription of the system

An open (flow) system : the system in which


material is transferred across the system
boundary.

Closed (batch) system : the system in which there


is no material transfer during the time interval of
interest.

By Assist. Prof. Dr. Wipada Sanongraj 44


1304 211 Chemical Engineering Principles
and Calculations

Closed system

System boundary

Open system

Mass in Mass out

System boundary

General Material Balance Equation

é Accumulation ù é Input through ù é Output through ù


ê within the system ú = êsystem boundary ú - êsystem boundary ú
ë û ë û ë û
éGaneration ù é Consumption ù
+ êê within ú - ê within
ú ê
ú
ú
êë the system úû êë the system úû

Material balance can refer to a balance on a system


for the

1. Total mass 2. Total moles


Units of the above equation must be mass or moles per time

By Assist. Prof. Dr. Wipada Sanongraj 45


1304 211 Chemical Engineering Principles
and Calculations

The accumulation term refers to a change in mass


or moles (plus or minus) within the system with
respect to time

The transfer through the system boundaries refers


to inputs to and outputs of the system.

The generation and consumption term refer to the


gain or loss of the mass or moles of the interesting
compound in the system.

Unsteady state : The values of the variables within


the system change with time.

Steady state : The values of the variables within the


system do not change with time.
Therefore, general material balance will be
simplified as:
é Accumulation ù é Input thorugh ù éOutput through ù
ê within the systemú = ê system boundary ú - ê system boundary ú
ë û ë û ë û
éGaneration ù éConsumption ù
+ êê within ú - ê within
ú ê
ú
ú
êëthe system úû êëthe system úû

By Assist. Prof. Dr. Wipada Sanongraj 46


1304 211 Chemical Engineering Principles
and Calculations

Example 2.1 : Total Mass Balance

A thickener in a waste disposal unit of a plant


removes water from wet sludge. How many
kilograms of water leave the thickener per 100 kg of
wet sludge that enter the thickener? The process is
in the steady state.

100 kg 70 kg
Thickener
Wet sludge Dehydrated sludge

Water = ?

First, we have to write the general mass balance.

é Accumulation ù é Input thorugh ù éOutput through ù


ê within the systemú = ê system boundary ú - ê system boundary ú
ë û ë û ë û
éGaneration ù éConsumption ù
+ êê within ú - ê within
ú ê
ú
ú
êëthe system úû êëthe system úû

Since there are no generation and consumption, and the


system is under steady state therefore the third and the last
terms on the right hand side and the term on the left hand
side equal to zero.

By Assist. Prof. Dr. Wipada Sanongraj 47


1304 211 Chemical Engineering Principles
and Calculations

é Input thorugh ù éOutput through ù


0=ê ú - ê system boundary ú
ë system boundary û ë û

100 kg = 70 kg + kg of water

kg of water = 30 kg

Example 2.2 Mass Balance

From the picture below, what is the accumulation


in the system? If there are no generation and
consumption.

Tank

Qin, Cin Qout, Cout

By Assist. Prof. Dr. Wipada Sanongraj 48


1304 211 Chemical Engineering Principles
and Calculations

é Accumulation ù é Input thorugh ù éOutput through ù


ê within the systemú = ê system boundary ú - ê system boundary ú
ë û ë û ë û
éGaneration ù éConsumption ù
+ êê within ú - ê within
ú ê
ú
ú
êëthe system úû êëthe system úû

Input = QinCin Output = QoutCout

Accumulation = Vtank(dC/dt)

é Accumulation ù dC
ê within the system ú = Vtan k dt = [QinCin ] - [Qout Cout ] + [ 0] - [ 0]
ë û

Problems

1. Draw a sketch of the following processes and


place a dashed line around the system:

a) Tea kettle

b) Fireplace

c) Swimming pool

By Assist. Prof. Dr. Wipada Sanongraj 49


1304 211 Chemical Engineering Principles
and Calculations

2. Label the materials entering and leaving the


systems in problem 1. Classify each system as
open or closed.

3. Write down the general material balance in


words. Simplify it for each process in problem 1,
stating the assumptions made in each
simplification.

Strategy for Analyzing Material Balance


Problems
v Read the problem and clarify what is to be
accomplished
vDraw a sketch of the process; define system by a
boundary
vLabel with symbols the flow of each stream and
compositions
vPut all the known values of compositions and
stream flows on the figure
vSelect a basis
vWrite down an appropriate set of balances to solve
vcount the number of independent balances
vSolve the equation
vCheck answers

By Assist. Prof. Dr. Wipada Sanongraj 50


1304 211 Chemical Engineering Principles
and Calculations

Solving Material Balance Problems That Do


Not Involve Chemical Reactions

Example 2.3 : Membrane Separation

Membranes represent a relatively new technology


for the separation of gases. The following figure
illustrates a nanoporous membrane that is made
by coating a very thin layer of the polymer on a
porous graphite-supporting layer. What is the
composition of the waste stream if the stream
amount to 80% of the input?

High-pressure Membrane Low-pressure


side side

21% O2
25% O2
Input
Flow Output
79% N2 75% N2

O2 N2
waste stream

By Assist. Prof. Dr. Wipada Sanongraj 51


1304 211 Chemical Engineering Principles
and Calculations

solution
This is a steady state process without chemical
reaction so that the accumulation term and the
generation and consumption terms are zero. The
system is the membrane. Let xO2 be the mole
fraction of oxygen and xN2 be the mole fraction of
nitrogen, and let nO2 and nN2 be the respective
moles.

F (g mol) P (g mol)
Membrane Mol fr.
Mol fr. O2 0.25
O2 0.21 W (g mol) N2 0.75
N2 0.79 Mol fr. Mol 1.00
1.00 O2 xO2 nO2
N2 xN2 nN2
1.00 W

Basis : 100 g mol = F

We know W = 0.80*100 = 80 g mol (because waste


amounts to 80% of input)

Three unknowns exist : P, xO2, and xN2 or P, nO2,


and nN2

Two independent balances are the oxygen and


nitrogen balances either as elements or as
compounds. The third independent balance is xO2
+ xN2 =1.00 or nO2 + nN2 = 80

By Assist. Prof. Dr. Wipada Sanongraj 52


1304 211 Chemical Engineering Principles
and Calculations

The component balances are


In Out
O2: 0.21(100) = 0.25(P) + xO2 (80)
N2: 0.79(100) = 0.75(P) + xN2 (80)

1.00 = xO2 + xN2

Or

In Out
O2: 0.21(100) = 0.25(P) + nO2
N2: 0.79(100) = 0.75(P) + nN2

80 = nO2 + nN2

The solution of these equations is xO2 = 0.20, xN2 =


0.80, and P = 20 gmol

A simpler calculation involves the use of the


total balance and one component balance
because
F= P+W or 100 = P + 80

Gives P = 20 straight off

By Assist. Prof. Dr. Wipada Sanongraj 53


1304 211 Chemical Engineering Principles
and Calculations

Example 2.4 Continuous Distillation

A manufacturer of alcohol is having a bit of


difficulty with a distillation column. Technicians
think too much alcohol is lost in the bottoms
(waste). Calculate the composition of the bottoms
and the mass of the alcohol lost in the bottoms.
Vapor
Heat Cooling water
Exchanger
1000 kg Feed (F)
10% EtOH Reflux
Distillate (Product) P = kg
90% H2O Distillation 60% EtOH
column 40% H2O
Bottom (B) kg Wt = 1/10 feed
Heat EtOH = ?
H2O = ?

General Mass Balance Equation:


é Accumulation ù é Input thorugh ù éOutput through ù
ê within the systemú = ê system boundary ú - ê system boundary ú
ë û ë û ë û
éGaneration ù éConsumption ù
+ êê within ú - ê within
ú ê
ú
ú
êëthe system úû êëthe system úû

Assuming that the process is in the steady state, no


reaction occurs
Mass in = Mass out
Basis : F = 1000 kg
Total mass balance; F = P + B (1)

By Assist. Prof. Dr. Wipada Sanongraj 54


1304 211 Chemical Engineering Principles
and Calculations

We are given that P is 1/10 of F, so that P =


0.1*1000 = 100 kg.

Substituting the total mass balance F = P+ B


calculate B by direct subtraction

B = 1000 – 100 = 900 kg

The solution for the composition of the bottom can


be computed directly by subtraction
kg feed in kg distillate out kg bottom out %
EtOH 0.1(1,000) - 0.6 (100) = 40 4.4

H2O 0.9(1,000) - 0.4 (100) = 860 95.6


900 100.0

Example 2.5 Mixing


Dilute sulfuric acid has to be added to dry charged batteries
at service station to activate a battery. You are asked to
prepare a batch of new 18.63% acid as follows. A tank of old
weak battery acid solution contains 12.43% H2SO4 (the
remainder is pure water). If 200 kg of 77.7% H2SO4 is added
to the tank, and the final solution is to be 18.63% H2SO4,
how many kilograms of battery acid have been made?
Added solution 200 kg = A

H2SO4 77.7%
H2O 22.3%
System

H2SO4 12.43% H2SO4 18.63%


H2O 87.57% H2O 81.37%

Original solution F kg Final solution P kg

By Assist. Prof. Dr. Wipada Sanongraj 55


1304 211 Chemical Engineering Principles
and Calculations

Should the process be treated as an unsteady-state


process or a steady state process?

If the tank is the system, and the tank initially contains


sulfuric acid solution, then a change occurs inside the
system so that accumulation occurs in the system (the
mass increases).

The general mass balance can be reduced to

Accumulation = In - Out

Basis : 200 kg of A

The two unknown quantities are F and P. We can write


two independent mass balances.

Type of Balance Accumulation in Tank A in Out

Final Initial

H2SO4 P(0.1863) - F(0.1243) = 200(0.777) - 0

H2O P(0.8137) - F(0.8757) = 200(0.223) - 0

Total P - F = 200 - 0

Because the equations are linear and only two occur, you
can take the total mass balance, solve it for F, and
substituting for F in the H2SO4 balance to calculate P.

(P-200)(0.1243) + 200(0.777) = P(0.1863)

P = 2110 kg acid
F = 1910 kg acid

By Assist. Prof. Dr. Wipada Sanongraj 56


1304 211 Chemical Engineering Principles
and Calculations

The problem could also be solved by considering the


mixing to be a steady process with the initial solution F
added to A in a vessel, and the resulting mixture removed
from the vessel.

A in F in P out

H2SO4 200(0.777) + F(0.1243) = P(0.1863)

H2O 200(0.223) + F(0.8757) = P(0.8137)

Total A + F = P

You can see by inspection that these equation are no


different than the first set of mass balances except for the
arrangement.

Example 2.6 Drying


In the processing of the fish, after the oil is
extracted, the fish cake is dried in rotary drum
dryers, finely ground, and packed. The resulting
product contains 65% protein. In a given batch of
fish cake that contains 80% water (the remainder is
dry cake), 100 kg of water is removed, and it is
found that the fish cake is then 40% water.
Calculate the weight of the fish cake originally put
into the dryer.

By Assist. Prof. Dr. Wipada Sanongraj 57


1304 211 Chemical Engineering Principles
and Calculations

W = 100 kg H2O
0 kg H2O

A kg
Rotary dryer
Wet fish Cake = ?
B kg
0.8 H2O Dry fish Cake = ?
0.2 BDC
0.4 H2O
0.6 BDC
Tie component

Basis : 100 kg of water evaporated = W

The unknown stream flows are two: A and B. All the


compositions are known.

Two independent balances can be written so that a


unique solution exists.
The water balance and the total mass balance plus the BDC
balance

The water balance:

0.8A = 0.4B + 100

We will use total mass balance plus the BDC balance rather
than the water balance. Because the BDC balance is
slightly easier to use. The water balance can be used as a
check on the calculations.

By Assist. Prof. Dr. Wipada Sanongraj 58


1304 211 Chemical Engineering Principles
and Calculations

In Out

Total balance A = B + W = B +100

BDC balance 0.2A = 0.6B

The BDC balance gives the ratio of A to B : B = 1/3A.


Introduce this relation into the total mass balance to get

A = 150 kg initial cake

Check via water balance:

0.8(150) = 0.4(150)(1/3) + 100

Example 2.7 Crystallization


A tank hold 10,000 kg of a saturated solution
of Na2CO3 at 30 oC. You want to
crystallize from this solution 3000 kg of
Na2CO3.10 H2O without any
accompanying water. To what
temperature must the solution be cooled?

By Assist. Prof. Dr. Wipada Sanongraj 59


1304 211 Chemical Engineering Principles
and Calculations

Saturated solution at ? oC
Saturated solution at 30 oC

Na2CO3 Na2CO3

H2O
H2O

System boundary
Na2CO3.10H2O
3000 kg

Solution
We need solubility data for Na2CO3 as a function of
temperature

Temp (oC) Solubility (g Na2CO3/100 g H2O)


0 7
10 12.5
20 21.5
30 38.8
At initial, solution is saturated at 30 oC, composition
of initial solution:
38.8 g Na2CO3/(38.8 g Na2CO3 + 100 g H2O)
= 0.280 mass fraction of Na2CO3 and 0.720 mass
fraction H2O

By Assist. Prof. Dr. Wipada Sanongraj 60


1304 211 Chemical Engineering Principles
and Calculations

Next, calculate the composition of crystals.


Basis: 1 g mol Na2CO3.10 H2O
Comp. Mol MW mass mass fr
Na2CO3 1 106 106 0.371
H2O 10 18 180 0.629
286 1.00
Basis: 10,000 kg of saturated solution at 30 oC

10,000 kg
Na2CO3 = 0.280 Na2CO3 P = ? Kg
Na2CO3
H2O = 0.720 mNa2CO3
mH2O
H2O
H2O

System 3,000 kg
Na2CO3.10H2O Na CO = 0.371
boundary 2 3
H2O = 0.629

System is unsteady state, mass balance equation reduces to


Accumulation = - Out
Mass balance for final state: mNa2CO3 + mH2O = P
Component balance:
Accumulation in tank Out
Final Initial
Na2CO3 mNa2CO3 - 10,000(0.28) = -3000(0.371)
H2O mH2O - 10,000(0.72) = -3000(0.629)
Total P - 10,000 = -3000

Solve equations get the compositions of final solution:


mNa2CO3 = 1687 kg, mH2O = 5313 kg, P = 7000 kg
To find temperature of final solution, calculate composition in terms of
g Na2CO3/ 100 g H2O
1687g Na2CO3/ 5313 g H2O = 31.8 g Na2CO3/ 100 g H2O
Do the interpolation from solubility data
30 oC – (38.8-31.8)/(38.8-21.5)*(30-20) oC = 26 oC

By Assist. Prof. Dr. Wipada Sanongraj 61


1304 211 Chemical Engineering Principles
and Calculations

Solving Material Balance Problems that


Involve Chemical Reactions

Objectives of this section

• Define flue gas, stack gas, Orsat analysis, dry


basis, wet basis, theoretical air (oxygen), required
air (oxygen), and excess air (oxygen).
• Given two of the three factors: entering gas
(oxygen), excess air (oxygen), and required air
(oxygen), compute the third factor.
• Understand how to apply the material balance
equation when chemical reactions occur.
• Apply the 10-step strategy to solve problems
involving reactions.

Flue or stack gas : All the gases resulting from a


combustion process including the water vapor,
sometimes known as wet basis.

Orsat analysis or dry basis : all the gases resulting


from the combustion process not including the
water vapor. Another way to state that water vapor
is not included in the gas analysis is to give the
analysis on a dry basis or give the Orsat analysis.

Theoretical air : the amount of air (or oxygen)


required to be brought into the process for
complete combustion. Sometimes this quantity is
called the required air.

By Assist. Prof. Dr. Wipada Sanongraj 62


1304 211 Chemical Engineering Principles
and Calculations

Excess air : the amount of air (or oxygen) in


excess of that required for complete combustion
as computed for theoretical air.

CO2
CO Flue gas
on SO2 Flue gas
Stack Gas O2
free basis or Orsat analysis
N2
or dry basis
SO2
H2O

Comparison of gas analyses on different bases

The calculated amount of excess air does not


depend on how much material is actually burned
but what can be burned. Even if only partial
combustion takes place, as for example, C
burning to both CO and CO2, the excess air is
computed as if the process of combustion
produced only CO2.

By Assist. Prof. Dr. Wipada Sanongraj 63


1304 211 Chemical Engineering Principles
and Calculations

1 C 1 CO2
Combustion
1 O2

C + O2 CO2
The material balance for O2 in moles
Acc. In Out Gen. Cons.
O2 0 = 1 - 0 + 0 - 1

The mole balances on C and CO2


C
O

The mass balances on C and CO2 (elements)

Acc. In Out Gen. Cons.


C
O

C
O2

By Assist. Prof. Dr. Wipada Sanongraj 64


1304 211 Chemical Engineering Principles
and Calculations

excess air excess O2 / 0.21


% excess air = 100 = 100
required air required O2 / 0.21

O2 entering process - O2 required


% excess air = 100
O2 required

excess O2
% excess air = 100
O2 entering - excess O2

Example 2.8 Excess Air


Fuel for motor vehicles other than gasoline
are being used because they generate
lower levels of pollutants than does
gasoline. Compressed propane has been
suggested as a source of economic power
for vehicles. Suppose that in a test 20 kg
of C3H8 is burned with 400 kg of air to
produce 44 kg of CO2 and 12 kg of CO.
What was the percent excess air?

By Assist. Prof. Dr. Wipada Sanongraj 65


1304 211 Chemical Engineering Principles
and Calculations

Solution
This is a problem involving the following reaction
C3H8 + 5O2 3CO2 + 4H2O

Since the percentage of excess air is based on the


complete combustion of C3H8 to CO2 and H2O

The required O2 is
20 kg C3H8 1 kg mol C3H8 5 kg mol O2
44 kg C3H8 1 kg mol C3H8

= 2.27 kg mol O2

The entering O2 is

400 kg air 1 kg mole air 21 kg mole O2 = 2.9 kgmole O


2
29 kg air 100 kg mole air

The percent excess air is

100 * excess O2 = 100* entering O2 – required O2


required O2

% excess air = 2.9 kg mole O2 – 2.27 kg mole O2 * 100


2.27 kg mole O2

= 28%

By Assist. Prof. Dr. Wipada Sanongraj 66


1304 211 Chemical Engineering Principles
and Calculations

Example 2.9
Suppose a gas containing 80% C2H6 and 20%
O2 is burned in an engine with 200%
excess air. Eighty percent of the ethane
goes to CO2, 10% goes to CO, and 10%
remained unburned. What is the amount
of excess air per 100 moles of the gas?

C2H6 + 7/2O2 2CO2 + 3H2O

Basis: 100 moles of gas

80 moles of C2H6 require 3.5(80) = 280 moles of O2,


However the gas contains 20 moles of O2, so that only
280-20 = 260 are needed in the entering air for complete
combustion. Thus the excess 200% excess O2 (air) is
based on 260 moles of O2
Entering with air Moles O2
required O2: 260
Excess O2 2(260)= 520
Total O2 3(260) = 780

By Assist. Prof. Dr. Wipada Sanongraj 67


1304 211 Chemical Engineering Principles
and Calculations

Example 2.10: Preventing corrosion


Corrosion of pipes in boilers by oxygen can be
alleviated through the use of sodiumsulfite.
Na2SO3 removes O2 from boiler feed water by
the following reaction:

2Na2SO3 + O2 2Na2SO4

How many pounds of Sodium sulfite are


theoretically required (for complete reaction) to
remove the oxygen from 8,330,000 lb of water
(106 gal) containing 10.0 ppm of dissolved
oxygen and at the same time maintain a 35%
excess of sodium sulfite?

H2O 8,330,000 lb H2O 8,330,000 lb


System
10 ppm O2 No oxygen

Basis: 8,330,000 lb H2O


The amount of O2 entering is
8,330,000 lb H2O 10 lb O2
= 83.3 lb O2
(1,000,000-10 lb O2) lb H2O
The O2 balance in lb is simple
In - Out + Generation – Consumption = Accumulation
83.3 – 0 + 0 – mO2 = 0
mO2 = 83.3 lb, then calculate the amount of Na2SO3

83.3 lb O2 1 lb mol O2 2 lb mol Na2SO3 126 lb Na2SO3 1.35


32 lb O2 1 lb mol O2 1 lb mol Na2SO3

= 886 lb Na2SO3

By Assist. Prof. Dr. Wipada Sanongraj 68


1304 211 Chemical Engineering Principles
and Calculations

Example 2.11 Combustion


Generation of methane-rich biogas is a way to
avoid high waste-disposal costs, and burning it
can meet up to 60% of the operating costs for
such waste-to-energy plants. Consider the
combustion of methane as shown in figure.

CH4 100% P = ? (kg mol)


F = 16 kg Burner CO2 = ?
A = 300 kg N2 = ?
O2 = 21% O2 = ?
N2 = 79% H2O = ?
300 kg A 1 kg mol A = 10.35 kg mol A in
29 kg A

16 kg CH4 1 kg mol CH4 = 1 kg mol CH4 in


16 kg CH4

10.35 kg mol A 0.21 kg mol O2 = 2.17 kg mol O2 in


1 kg mol A

10.35 kg mol A 0.79 kg mol N2 = 8.18 kg mol N2 in


1 kg mol A

By Assist. Prof. Dr. Wipada Sanongraj 69


1304 211 Chemical Engineering Principles
and Calculations

Basis: 16 kg CH4 = 1 kg mol CH4


The unknowns are P and the four compositions in P:
nPCO2, nPO2, ,nPN2, and nPH2O.
Let use the balance on the elements.
Balance CH4 in Air in Out
C: 1 = nPCO2
H2: 2 = nPH2O
O2: 2.17 = 0.5nPH2O+nPO2 + nPCO2
N2: 8.18 = nPN2
nPCO2 + nPH2O + nPN2 + nPO2 = P

Now we can easily solve the set of equations;

nPCO2 = 1
nPH2O = 2
nPN2 = 8.18
nPO2 = 2.17 - 0.5(2) – 1 = 0.17
P = 1 + 2 + 8.18 + 0.17 = 11.35
From xiP = (niP)/P
xPCO2 = 0.09 xPN2 = 0.72
xPH2O = 0.18 xPO2 = 0.01

By Assist. Prof. Dr. Wipada Sanongraj 70


1304 211 Chemical Engineering Principles
and Calculations

Example 2.12: Combustion with Nonprecise Data


In a test run, a liquid that is proposed for use as a
fuel in a flare and has the composition of 88% C and
12% H2 is vaporized and burned with dry air to a flue
gas of the following composition on a dry basis.
CO2 13.4%
O2 3.6%
N2 83.0%
100.0%
To help design the equipment of the continuous
steady-state combustion device, determine how
many kg mol of dry flue gas are produced per 100
kg of liquid feed. What was the percentage of
excess air used?

The process is in the steady-state accompanied


by chemical reaction. Let the system be the
flare and associated equipment. HO 1.00
2

W
H2O
kg mol

mass fr.
F Catalytic G
C 0.88
Test liquid oxidation unit kg mol
H2 0.12
kg
1.00

mol fr.
mol fr. A CO2 0.134
N2 0.79 Air (dry) N2 0.830
O2 0.21 kg mol O2 0.036
1.00 1.00

By Assist. Prof. Dr. Wipada Sanongraj 71


1304 211 Chemical Engineering Principles
and Calculations

Basis : 100 kg mol dry flue gas = G


The atomic species balances (in moles)
Balance F in A in W out G out Accum.

C: 0.88F + 0 - 0 - 0.134(100) = 0
12
H2: 0.12F + 0 - W - 0 = 0
2.016

O2: 0 + 0.21A - (1/2)W - (0.134 +0.036)100 = 0

N2: 0 + 0.79A - 0 - (0.830)100 =0

From C : F = (1304*12/0.88) = 182.73 kg

From N2 : A = 83.0/0.79 = 105.06 kg mol

From H2 : W = 182.73*0.12/2.016 = 10.88 kg mol

Use the oxygen balance as a check

105.06(0.21) = 10.88*(1/2) + 17.00


22.06 = 22.44
An exact balance does not occur, but the answers agree
reasonably well here. In many combustion problems,
slight errors in the data will cause large differences in the
calculated flows and percentage of excess air.

By Assist. Prof. Dr. Wipada Sanongraj 72


1304 211 Chemical Engineering Principles
and Calculations

We can now answer the requested questions


G 100 kgmol 54.73 kgmolG
= =
F 182.73 kg 100 kgF
The percent excess air can be calculated via two routes:

excess O2
% excess air = 100
O2 entering - excess O2

or excess O2
% excess air = 100
required O2

The required O2 is
C + O2 CO2 182.73*(0.88) = 13.40 kgmol
12

H2 + 1/2O2 H2O 182.73*(0.12) = 5.44 kgmol


2(2.106)

The overall required O2 is 18.84 kgmol

The excess oxygen is


22.4 - 18.84 = 3.56
or
22.06 - 18.84 = 3.22

By Assist. Prof. Dr. Wipada Sanongraj 73


1304 211 Chemical Engineering Principles
and Calculations

Basis: O2 in = 22.44 mol Basis: O2 in = 22.06 mol


calculated from the calculated from the
entering air flue gas

% excess air: 100*3.60 = 19.1% 100*3.22 = 16.7%


22.44-3.60 22.06-3.22

% excess air: 100*3.60 = 19.1% 100*3.22 = 16.7%


18.84 18.84

Example 2.13 : Combustion of Coal


A local utility burns coal having the following
compositions on a dry basis.

Component Percent
C 83.05
H 4.45
O 3.36
N 1.08
S 0.70
Ash 7.36
Total 100.0

By Assist. Prof. Dr. Wipada Sanongraj 74


1304 211 Chemical Engineering Principles
and Calculations

The average Orsat analysis of the gas from the


stack during a 24-hr test was

Component Percent
CO2 + SO2 15.4
CO 0.0
O2 4.0
N2 80.6
Total 100.0
Moisture in the fuel was 3.9%, and the air on the average
contained 0.0048 lb H2O/lb dry air. The refuse showed
14.0% unburned coal, with the remainder being ash. You are
asked to check the consistency of the data before they are
stored in a data base. Is the consistency satisfactory? What
was the average percent excess air used?

This is a steady-state problem with the reaction.


The system is the furnace. %
CO2+SO2 15.4
lb W (lbmol) H2O(g) 100% CO 0.0
C 83.05 O2 4.0
H 4.45 Coal Stack gas N2 80.6
O 3.36 F (lb) furnace P (lb mol) 100.0
N 1.08
S 0.70 Air A
Ash 7.36 (lb mol) Refuse R (lb) %
100.00 C + H + O +N + S 14.0
Mol fr.
Added H2O 3.9 lb N2 0.79 Ash 86.0
H: 0.433 lbmol O2 0.21 100.0
O: 0.217 lbmol 1.00
Added H2O 0.0048 lb/lbAir
H: 0.0154 lbmol/lb molA
O: 0.0077 lbmol/lb mol A

By Assist. Prof. Dr. Wipada Sanongraj 75


1304 211 Chemical Engineering Principles
and Calculations

Basis : F = 100 lb

We might neglect the C, H, O, N, and S in the


refuse but will include the amount to show what
calculations are necessary if the amounts of the
elements were significant. The ash balance is:

7.36 = R(0.86)
R = 8.56 lbs

The unburned coal in the refuse is:

8.56(0.14) = 1.20 lb

If we assume that the combustibles in the refuse


occur in the same proportions as they do in the coal
(which may not be true), the quantities of the
combustibles in R on an ash-free basis are:

Component mass % lb lb mol


C 89.65 1.076 0.0897
H 4.80 0.058 0.0537
O 3.63 0.0436 0.0027
N 1.17 0.014 0.0010
S 0.76 0.009 0.0003
Total 100.00 1.20 0.1474

The variables whose values are still unknown are


A, W, and P

By Assist. Prof. Dr. Wipada Sanongraj 76


1304 211 Chemical Engineering Principles
and Calculations

Mass balances of the elements in moles


In Out
F A W P R
C+S 83.05 + 0.70 + 0 = 0 + P(0.154) + 0.0897 + 0.0003
12.0 32.0
H 4.45 + 0.433 + 0.0154A = 2W + 0 + 0.0537
1.008
0 3.36 + 0.217 + 0.21A(2) + 0.0077A = W + 2P(0.154+0.04) + 0.0027
16.0

N 1.08 + 2(0.79A) = 0 + 2P(0.806) + (0.001)


14.0
Solve the C+S balance to get P = 50. Then solve the N
balance to get A = 45.35. Solve the H balance to get W =
2.746.

Use the O balance to serve as a check: 19.8 =20.0

To calculate the excess air, because of the


oxygen in the coal and the existence of unburned
combustibles, we will calculate the total oxygen in
and the required oxygen:

O2 entering process - O2 required


% excess air = 100
O2 required

By Assist. Prof. Dr. Wipada Sanongraj 77


1304 211 Chemical Engineering Principles
and Calculations

Component Reaction lb lb mol Required


O2 (lb mol)

C C + O2 CO2 83.05 6.921 6.921


H H2 + 1/2O2 H2O 4.45 4.415 1.104

O - 3.36 0.210 (0.105)

N - - - -

S S + O2 SO2 0.70 0.022 0.022


7.942

The oxygen in the air is 45.35*0.21 = 9.524 lbmol.

9.524 - 7.942
% excess air = 100 = 19.9%
7.942

If you (incorrectly) calculated the % excess air


from the wet stack gas alone, you would get

4.00
% excess air = 100 = 23.8%
15.4 + 2.746 / 2

By Assist. Prof. Dr. Wipada Sanongraj 78


1304 211 Chemical Engineering Principles
and Calculations

Recycle, Bypass, and Purge Calculations


A recycle stream is a term denoting a process
stream that returns material from downstream of a
process unit back to the process unit.

A bypass stream is the one that skips one or more


stages of the process and goes directly to another
down stream stage.

A purge stream is a stream bled off to remove an


accumulation of inert or unwanted material that
might otherwise build up in the recycle stream.

Recycle, R

Fresh Process Gross Net


Mixer Feed Process Separator Product, P
Feed, F Product

2 4
3
1
1. About the entire process
2. About the junction point at which the fresh feed
is combined with the recycle stream
3. About the process only
4. About the function point at which the gross
product is separated into recycle and net product

By Assist. Prof. Dr. Wipada Sanongraj 79


1304 211 Chemical Engineering Principles
and Calculations

Example 2.14 : Recycle without Chemical Reaction


A distillation column separates 10,000 kg/hr of a
50% benzene-50% toluene mixture. The product D
recovered from the condenser at the top of the
column contains 95% benzene, and the bottom W
from the column contains 96% toluene. The vapor
stream V entering the condenser from the top of the
column is 8000 kg/hr. A portion of the product from
the condenser is returned to the column as reflux,
and the rest is withdraw for use elsewhere. Assume
that the compositions of the streams at the top of
the column (V), the product withdrawn (D), and the
reflux (R) are identical because the V stream is
condensed completely. Find the ratio of the amount
refluxed R to the product withdrawn (D).

Basis : 1 hr (equal F = 10,000 kg)


Overall Material Balances:
Total material
F=D+W
10,000 = D + W
Component (benzene)
FwF = DwD + WwW
10,000(0.50) = D(0.95) + W(0.04)
Solving for W and D
W = 4950 kg/hr
D = 5050 kg/hr

By Assist. Prof. Dr. Wipada Sanongraj 80


1304 211 Chemical Engineering Principles
and Calculations

Balance around the condenser


Total material:
V=R+D
8,000 = R + 5,050
R = 2,950 kg/hr
R/D = 2950/5050 = 0.58

Example 2.15 : Recycle without chemical reaction


The manufacture of such products as penicillin,
tetracycline, vitamins, and other fine organic
compounds, usually requires separating the
suspended solids from their mother liquor by
centrifuging, and then drying the wet cake. What is
the lb/hr of the recycle stream R?

This is a steady-state problem without reaction and


with recycle.

By Assist. Prof. Dr. Wipada Sanongraj 81


1304 211 Chemical Engineering Principles
and Calculations

R=?
0.286V
0.714 H2O

F
98 lb/hr Continuous
20% V Centrifuge
80% H2O Filter

C=?
60% V
40% H2O
P=?
W=? 4% H2O
H2O 100% 96 % V

Basis : 1 hr (F = 98 lb)
Overall mass balances
In Out
V: 0.20(98) = 0 + 0.96P
H2O : 0.80(98) = (1.0)W + 0.04P
Total : 98 = W + P
Solving for P and W
P = 20.4 lb W = 98 – 20.4 = 77.6 lb

By Assist. Prof. Dr. Wipada Sanongraj 82


1304 211 Chemical Engineering Principles
and Calculations

Total balance on filter


C=R+P
C = R + 20.4
Component V balance on filter
CwC = RwR + PwP
0.6C = 0.286R + 0.96(20.4)
Solving for R
R = 23.4 lb/hr

Recycle in Processes with


Chemical Reaction
Overall fraction conversion
[Mass (moles) of reactant in fresh feed – mass
(moles) of reactant in output of the overall process]
/ mass (moles) of reactant in fresh feed
Single-pass (once through) fraction conversion
[Mass (moles) of reactant fed into the reactor
(process feed) – mass (moles) of reactant existing
the reactor (gross product)]/ mass (moles) of
reactant fed into the reactor

By Assist. Prof. Dr. Wipada Sanongraj 83


1304 211 Chemical Engineering Principles
and Calculations

Process feed & Gross Product

Recycle

Gross product
Fresh Feed Process feed

Net product

Example 2.16 : Recycle with a Reaction Occurring


Immobilized glucose isomerase is used as a
catalyst in producing fructose from glucose in a
fixed-bed reactor (water is the solvent). For the
system shown below, what percent conversion of
glucose results on one pass through the reactor
when the ratio of the exit stream to the recycle
stream in mass units is equal to 8.33?
R (kg)
wR,G = ?
wR,F = ?
S = 100 kg wR,W = ?
0.4ws,G 1.00 P = ? (kg)
0.6ws,,w wR,G = ?
1.00
T (kg) Fixed-bed wR,F = ?
1 wT,G = ? 2 wR,W = ?
Reactor
wT,F = 0.04 1.00
wT,W = ?

By Assist. Prof. Dr. Wipada Sanongraj 84


1304 211 Chemical Engineering Principles
and Calculations

Basis : S = 100 kg

Let f be the fraction conversion for one pass through


the reactor. The unknowns are R, F, P, T, wR,G, wR,T,
wR,W, wT,W, wT,G, and f for a total of 9.
The balances are SwR,i = 1, SwT,i = 1, R = P/8.33
Overall balances:
Total : S = P = 100 kg
R = 100/8.33 = 12 kg
No water is generated or consumed
Water : 100(0.60) = P(wR,W) = 100wR,W
wR,W = 0.6

Mixing point 1, no reaction occurs


Total : 100 + 12 = T = 112
Glucose: 100(0.40) + 12(wR,G) = 112(wT,G)
Fructose: 0 + 12(wR,F) = 112(0.04)
wR,F = 0.373
Because wR,F + wR,G + wR,W = 1
wR,G = 1 – 0.373 – 0.600 = 0.027
And from the glucose balance,
wT,G = 0.360

By Assist. Prof. Dr. Wipada Sanongraj 85


1304 211 Chemical Engineering Principles
and Calculations

Reactor plus Separator 2


Total : T = 12 + 100 = 112
Glucose :
In Out Consumed Accumu.

TwT,G - (R+P)wR,G - fTwT,G = 0

112(0.360) – 112(0.027) - f(112)(0.360) = 0

f = 0.93

Example 2.17: Recycle with a reaction occurring

Refined sugar (sucrose) can be converted to glucose and


fructose by the inversion process
C12H22O11 + H2O = C6H12O6 + C6H12O6

The combined quantity glucose plus fructose is called


inversion sugar. If 90% conversion of sucrose occurs on
one pass through the reactor, what would be the recycle
stream flow per 100 lb fresh feed of sucrose solution
entering the process shown in Figure? What is the
concentration of inversion sugar (I) in the recycle stream
and in the product stream? The concentrations of
components in the recycle stream and product stream
are the same

By Assist. Prof. Dr. Wipada Sanongraj 86


1304 211 Chemical Engineering Principles
and Calculations

R
xs,R
xI,R
Xw,R
C P
F = 100 lb Separator
Sucrose 30% xs,C Reactor xs,R
H2O 70% xI,C = 0.05 xI,R
xw,C Xw,R

Solution
Basis: 100 lb = F
Overall balance: F = P = 100 lb
Balance at mixing point:
Total 100 + R = C (a)
Sucrose 100(0.3) + R(xs,R) = C(xs,C) (b)
Inversion 0 + R(xI,R) = C(0.05) (c)

Make balance on the reactor plus the separator


In Out Consumed
Sucrose C(xs,C) - (R+100)(xs,R) – C(xs,c)(0.9) = 0 (d)

Next do the water balance, but we need to calculate the


pounds of water consumed in the reaction per pound of
sucrose consumed
1 lbmol H2O 1 lbmol S 18 lb H2O = 0.0526 lb H2O
1 lbmol S 342.35 lb S 1 lbmol H2O lb S

In Out Consumed
Water Cxw,C - (R+100)xw,R - Cxs,C(0.9)(0.0526) (e)
Xs,R + xI,R + xw,R = 1 (f)
Xs,C + xI,C + xw,C = 1 (h)
Solve equations (a-h), we get R = 20.9 lb and xI,R = 0.313

By Assist. Prof. Dr. Wipada Sanongraj 87


1304 211 Chemical Engineering Principles
and Calculations

Bypass and Purge


A bypass stream- one that skips one or
more stages of the process and goes
directly to another downstream stage
A purge- a stream bled off to remove an
accumulation of inerts or unwanted
materials

Example 2.18: Bypass


Calculations

In the feedstock preparation section of a plant


manufacturing natural gasoline, isopentane is removed
from butane-free gasoline. Assume for purposes of
simplification that the process and components are as
shown in figure. What fraction of the butane-free
gasoline is passed through the isopentane tower? The
process is in the steady state and no reaction occurs.

By Assist. Prof. Dr. Wipada Sanongraj 88


1304 211 Chemical Engineering Principles
and Calculations

Isopentane side stream, S kg


i-C5H12 100 %

Iso-
De-
pentane
butanizer X kg Y kg
tower
n-C5H12 100%

F = 100 kg
n-C5H12 80% To natural gasoline plant, P kg
i-C5H12 20% n-C5H12 = 90%
i-C5H12 = 10%

From diagram part of the butane-free gasoline bypasses


the isopentane tower and proceeds to the next stage in
the natural gasoline plant.

Basis: 100 kg feed


Overall balance
Total material balance: In = Out
100 = S + P (a)
Component balance (n-C5H12)
100(0.8) = S(0) + P(0.9) (b)
We get, P = 88.9 kg and S = 11.1 kg
Balance around isopentane tower
Total material balance: x = 11.1 + y (c)
Component balance (n-C5H12)
x(0.8) = y(1) (d)
We get, x = 55.5 kg, y = 44.4 kg, therefore the fraction of
butane free gas passed through isopentane tower is
55.5/100 = 0.55

By Assist. Prof. Dr. Wipada Sanongraj 89


1304 211 Chemical Engineering Principles
and Calculations

Another approach is to make a balance at mixing


points 1 and 2

Balance around mixing point 2

Total material balance: (100-x) + y = 88.9 (e)


Component balance (i-C5H12): (100-x)0.2 = 88.9*0.1 (f)

We get, x = 55.5 kg

Example 2.19: Purge


Considerable interest exists in the conversion of coal into
more convenient liquid products for subsequent
production of chemicals. Two of the main gases that can
be generated are H2 and CO. After cleanup, these two
gases can be combined to yield methanol according to
the following equation
CO + 2H2 = CH3OH
Figure illustrates a steady state process. All compositions
are in the mole fractions or percent. The stream flows
are in moles. A purge stream is used to maintain the CH4
concentration in the exit to the separator at no more than
3.2 mol %. The once-through conversion of the CO in the
reactor is 18%. Compute the moles of recycle, CH3OH,
and purge per mole of feed and purge composition.

By Assist. Prof. Dr. Wipada Sanongraj 90


1304 211 Chemical Engineering Principles
and Calculations

F mol
67.3% H2 E mol
32.5% CO Reactor Separator 100% CH3OH
0.2% CH4

Recycle, R Purge, P mol


X H2
Y CO
Z CH4

Because the problem is presented in terms of moles,


making an overall mass balance is not convenient.
We will use Element balance instead.

Basis: F = 100 mol


x+y+z =1 (a)
From limit of CH4 in the reactor, z = 0.032 (b)

Overall element balance (in moles)


H2: 67.3 + 0.2(2) = E(2)(1) + P(x+2z) (c)
C: 32.5 + 0.2 = E(1) + P(z + y) (d)
O: 32.5 = E(1) + (y)P (e)

Reactor plus separator balance (in moles)


In Out Consumed
CO: 32.5 + R(y) - y(R+P) = (32.5+Ry)(0.18) (f)
Solve these equations we get
E = 31.25 moles, P = 6.25 moles, R = 705 moles, x = 0.768
y = 0.2, z = 0.032

By Assist. Prof. Dr. Wipada Sanongraj 91


1304 211 Chemical Engineering Principles
and Calculations

Chapter 3
Energy Balance

Introduction
System: 304 12 1 Chemi cal Engineering Principles
Closed system: lucose: 100(0.40) + 12 (
I00 = S + P
Open system: otal m ta er ila ba l ance: (100 / nerts
E y)
Property: Šr o perty:/a t er : 100(0.60) =
pen system:
Extensive Property: olve these equa t ions we get
utane fr e e ga s pas sed through
Intensive Property: olve these equt a ions we get
Šroperty:
State: Entroduction

By Assist. Prof. Dr. Wipada Sanongraj 92


1304 211 Chemical Engineering Principles
and Calculations

Six types of Energy


Work (W): form of
energy that represents s2
W = ò Fds
a transfer between the
system and
surroundings
s1
W is positive when
done on the system
F is external force in the direction of
s acting on the system

Example 3.1: Work


Suppose that an ideal gas at 300 K and 200 kPa is
enclosed in a cylinder by a frictionless piston, and the
gas slowly forces the piston so that the volume of gas
expands from 0.1 to 0.2 m3. Calculate the work done by
the gas on the piston if two different paths are used to go
from the initial state to the final state:
Path A: The expansion occurs at constant pressure (p =
200 kPa)
Path B: The expansion occurs at constant temperature
(T= 300 K)

By Assist. Prof. Dr. Wipada Sanongraj 93


1304 211 Chemical Engineering Principles
and Calculations

Solution
s2 v2
F
W = - ò Ads = - ò pdV
s1
A v1

Path A:
W = -p(V2-V1) = - (200*103 Pa)(0.1 m3) = -20 kJ

Path B:
v2 v2
nRT V
W = - ò pdv = - ò dV = -nRT ln( 2 )
v1 v1
V V1
n = (200 kPa)(0.1 m3)/(300 K)(8.314 kPa m3) = 0.00802 kg mol

W = (0.00802 kg mol)(8.314 kJ/kg mol K)(300 K) ln2 = -13.86 kJ

Heat
Part of total energy flow across a system boundary that
is caused by a temperature difference between the
system and the surrounding
Heat is positive when transferred to the system
Heat maybe transfer by conduction, convection, or
radiation
Q = UADT ; A is area for heat transfer, U is an empirical
.
coefficient, DT is the effective temperature difference
between the system and its surroundings, and is the
rate of heat transfer. Q&

By Assist. Prof. Dr. Wipada Sanongraj 94


1304 211 Chemical Engineering Principles
and Calculations

Kinetic Energy (K)


Energy caused by system’s velocity
relative to the surrounding at rest.
K = 1/2mv2
Specific kinetic energy (k) is the kinetic
energy per unit mass
k = K/m

Example 3.2 : Kinetic


Energy
Water is pumped from a storage tank into
a tube of 3.00 cm inner diameter at the rate
of 0.001 m3/s. See Figure. What is the
specific kinetic energy of the water?
3.00 cm ID

0.001 m3/s

By Assist. Prof. Dr. Wipada Sanongraj 95


1304 211 Chemical Engineering Principles
and Calculations

Solution
Basis: 0.001 m3/s of water
Assume that r = 1000 kg/m3
r = ½(3.00) = 1.50 cm
v = 0.001 m3 (100 cm)2
s p(1.50)2 cm2 (1 m)2
= 1.415 m/s

k = 1 (1.415)2 1 N 1J
2 (s/m)2 (1 kg m/s2) 1 Nm
= 1.0 J/kg

Potential Energy (P)


Energy caused by body force exerted on
its mass by a gravitational or
electromagnetic field with respect to a
reference surface
P = mgh
Specific potential energy (p) is normalized
by mass
p = P/m = gh

By Assist. Prof. Dr. Wipada Sanongraj 96


1304 211 Chemical Engineering Principles
and Calculations

Example 3.3: Potential


Energy
Water is pumped from one reservoir to
another 300 ft away. The water level in the
second reservoir is 40 ft above the water
level of the first reservoir. What is the
increase in specific potential energy of the
water in Btu/lbm?

Solution
300 ft

40 ft

Let the water level in the first reservoir be the reference plane, then h = 40 ft

p = 32.2 ft/s2 40 ft 1 Btu = 0.0514 Btu/lbm


32.2 lbm ft 778.2 ft lbf
lbf s2

By Assist. Prof. Dr. Wipada Sanongraj 97


1304 211 Chemical Engineering Principles
and Calculations

Internal Energy (U)


Energy of molecule, atom, and subatom.
Internal energy per unit mass (u) can be
calculated from measurable variables such
as pressure, volume, temperature, and
composition.

u = u (T , V )
æ ¶u ö æ ¶u ö ˆ
du = ç ÷ dT + ç ÷ dV
è ¶T øVˆ è ¶V ø T
ˆ

By definition (¶u / ¶T )Vˆ is the heat capacity at constant


volume and the second term on the right-hand side of
the above equation is so small so that it can be
neglected.

Changes in the internal energy can be computed by


the following equation:
T2

u 2 - u1 = ò C v dT
T1

By Assist. Prof. Dr. Wipada Sanongraj 98


1304 211 Chemical Engineering Principles
and Calculations

Enthalpy (H)
H = U + pV where p is the pressure and V is
the volume.

æ ¶Hˆ ö æ ¶Hˆ ö
dHˆ = çç ÷ dT + ç
÷
÷
ç ¶P ÷ dP
Ĥ = enthalpy per unit mass
è ¶T ø p è øT
T2
Hˆ 2 - Hˆ 1 = ò C p dT
(neglect second term of above
T1 eq. and (¶Hˆ / ¶T ) is the heat
p

capacity at constant pressure.)

However, in processes operating at high pressures,


the second term on the right hand side of the above
eq. cannot necessarily be neglected.

Calculation of Enthalpy
Changes
Phase transition enthalpy change latent heat
Enthalpy changes in a single phase: sensible heat
Enthalpy changes for the phase transition: heat of fusion
and heat of vaporization
Heat of condensation is the negative of the heat of
vaporization.
Solid changes directly to vapor: heat of sublimation

By Assist. Prof. Dr. Wipada Sanongraj 99


1304 211 Chemical Engineering Principles
and Calculations

Enthalpy Computing

Heat capacity equation


Tables
Enthalpy charts
Computer databases

By Assist. Prof. Dr. Wipada Sanongraj 100


1304 211 Chemical Engineering Principles
and Calculations

Heat Capacity Equations


T2

DH = ò C p dT
T1

Units: kJ/(kg mol)(K) or Btu/(lb mol)(F)


For ideal gas mixtures, Cpavg = SxiCpi
Cp = a + bT + cT2

Example 3.4: Heat Capacity


Equation
Heat capacity equation for CO2 gas is Cp =
2.675*104 + 42.27T – 1.425*10-2T2
Cp is in J/kg mol K and T is in K. Convert
this equation into a form so heat capacity
will be in Btu/lb mol F with T in F

By Assist. Prof. Dr. Wipada Sanongraj 101


1304 211 Chemical Engineering Principles
and Calculations

J 1 Btu 1DK 1DR 0.4536 kg


kg mol DK 1055 J 1.8DR 1DF 1 lb

Next, substitute the relation between the


temperature in K and the temperature in F
TK = TR/1.8 = (TF + 460)/1.8
Carry out the mathematical operations, get

Cp = 8.702 + 4.66*10-3TF – 1.053*10-6TF2

Example 3.5: Calculation of DH for a


gas mixture using heat capacity
equation
The conversion of solid wastes to gas can be
accomplished in incinerators. Solid waste can be burned
to a gas of the following composition (on dry basis)
CO2 9.2%, CO 1.5%, O2 7.3%, and N2 82%. What is the
enthalpy difference for this gas per lb mol between the
bottom and the top of the stack if the temperature at the
bottom of the stack is 550 F and temperature at the top is
200 F?

By Assist. Prof. Dr. Wipada Sanongraj 102


1304 211 Chemical Engineering Principles
and Calculations

Solution
The heat capacity equations are ( in Btu/lb mol F)
N2: Cp = 6.895 + 0.7624*10-3T – 0.7009*10-7T2
O2: Cp = 7.104 + 0.7851*10-3T – 0.5528*10-7T2
CO2: Cp = 8.448 + 5.757*10-3T – 21.59*10-7T2 + 3.059*10-10T3
CO: Cp = 6.865 + 0.8024*10-3T – 0.7367*10-7T2
Basis: 1 lb mol of gas
Multiplying these equations by the respective mole fraction of
each component and adding them together can save time in
integration.

N2:Cp = 0.82(6.895 + 0.7624*10-3T – 0.7009*10-7T2)


O2: Cp = 0.073(7.104 + 0.7851*10-3T – 0.5528*10-7T2)
CO2: Cp = 0.092(8.448 + 5.757*10-3T – 21.59*10-7T2 + 3.059*10-
10T3)

CO: Cp = 0.015(6.865 + 0.8024*10-3T – 0.7367*10-7T2)


Cpnet = (7.053 + 1.2242*10-3T – 2.6124*10-7T2 + 0.2814*10-10 T3)dT

200
DH = ò Cpnet dT
550

DH = -2468.6-160.7+13.8-0.633 = -2616 Btu/lb mol gas

By Assist. Prof. Dr. Wipada Sanongraj 103


1304 211 Chemical Engineering Principles
and Calculations

Example 3.6
Calculate the enthalpy change for 1 kg
mol of N2 gas that is heated at constant
pressure of 100 kPa from 18 to 1100 oC

Solution

Because 100 kPa is 1 atm, the enthalpy of combustion gas


(Appendix D.6) can be used.
At 1100 oC (1373 K): DH = 34,715 kJ/kg mol (by
interpolation)
At 18 oC (291 K): DH = 524 kJ/kg mol

For 1 kg mol of N2
DH = 34,715 – 524 = 34,191 kJ/kg mol

By Assist. Prof. Dr. Wipada Sanongraj 104


1304 211 Chemical Engineering Principles
and Calculations

Example 3.7
Steam is cooled from 640 oF and 92 psia to
480 oF and 52 psia. What is DH in Btu/lb?

Solution
Use steam table and do the double interpolation first
between pressures at fixed temperature, and then
between temperatures at fixed pressures.
Steps:
1st: Get DH at P = 50, 55, 90, and 95 psia at each
temperatures of 600 and 700 oF.
2nd: Interpolate between P = 50 and 55 psia and temp 600
oF to get DH at P = 52 psia and T = 600 oF

3rd: Interpolate between P = 90 and 95 psia and temp 600


oF to get DH at P = 92 psia and T = 600 oF

4th: Interpolate between T = 600 and 700 oF and P = 52


psia to get DH at T = 640 oF and P = 52 psia
5th: Interpolate between T = 600 and 700 oF and P = 92
psia to get DH at T = 640 oF and P = 92 psia

By Assist. Prof. Dr. Wipada Sanongraj 105


1304 211 Chemical Engineering Principles
and Calculations

An example of the interpolation needed at 600 oF is

2*(1328.7-1328.4)/5 = 0.12
So, at p = 92 psia and T = 600 oF , DH = 1328.7-0.12 = 1328.6
The enthalpy change is
DH = 1272.8 – 1348.4 = -75.6 Btu/lb

By Assist. Prof. Dr. Wipada Sanongraj 106


1304 211 Chemical Engineering Principles
and Calculations

Example 3.8
Four kilograms of water at 27 oC and 200
kPa are heated at constant pressure until
the volume of the water becomes 1000
times the original value. What is the final
temperature of the water?

Solution
The initial specific volume is the saturated liquid water
volume at 300 K = 0.001004 m3/kg.
The final volume is = 0.001004*1000 = 1.004 m3/kg
At 200 kPa, using interpolation between 400 and 450 K
which covers the range of the specific volume of 1.004
m3/kg, we find T by solving;
(T2-T1)(DV)/(V2-V1) = DT
0.9024 + (1.025-0.9024)/(450-400)*(T-400) = 1.004 m3/kg
T = 400 + 41 = 441 K

By Assist. Prof. Dr. Wipada Sanongraj 107


1304 211 Chemical Engineering Principles
and Calculations

General Energy Balance


{accumulation of energy within the
system} = {transfer of energy into system
through system boundary} – {transfer of
energy out of system through system
boundary} + {energy generation within
system} – {energy consumption within
system}

Energy Balance without


reaction for closed system
W=- W=+ P2
K2
K1
U2

U1 h2

h1
P1
Q=+ Q=-

DE = Et2 - Et1= DU+DP+DK = Q + W

General process showing the system boundary and


energy transport across the boundary.

By Assist. Prof. Dr. Wipada Sanongraj 108


1304 211 Chemical Engineering Principles
and Calculations

Energy Balances for open


system (without chemical
reaction)
Accumulation: DE = mt2(u+k+p)t2-mt1(u+k+p)t1
Energy transfer in with mass flow: (u1+k1+p1)m1
Energy transfer out with mass flow:
(u2+k2+p2)m2
Net transfer by heat flow in: Q
Net transfer by mechanical or electrical work in:
W
Net transfer by work to introduce and remove
mass: p1v1m1-p2v2m2

Application of the general


energy balance without
reaction
1) No mass transfer (closed or batch
system) (m1= m2=0): DE = Q+W
2) No accumulation (DE = 0), no mass
transfer (m1= m2= 0): Q = -W
3) No accumulation (DE = 0), but with mass
flow: Q+W = D(h+k+p)m
4) No accumulation, Q = 0, W = 0, k = 0, p =
0: DH = 0

By Assist. Prof. Dr. Wipada Sanongraj 109


1304 211 Chemical Engineering Principles
and Calculations

Special process names


associated with energy
balance
Isothermal (dT=0): constant-temperature process
Isobaric (dp = 0): constant pressure process
Isometric or isochoric (dV = 0): constant-volume
process
Adiabatic (Q = 0): no heat interchange (i.e., an
insulated system)
System is insulated
Q is very small in relation to the other terms
The process takes place so fast that there is no time
for heat to be transferred

Example 3.9 : Closed


system
10 lbs of CO2 at room temperature (80 oF)
are stored in a fire extinguisher having a
volume of 4 ft3. How much heat must be
removed from the extinguisher so that
40% of the CO2 becomes liquid?

By Assist. Prof. Dr. Wipada Sanongraj 110


1304 211 Chemical Engineering Principles
and Calculations

Solution
Closed system without reaction
Use CO2 chart in Appendix J to get properties
Specific volume of CO2 is 4/10 = 0.4 ft3/lb, hence CO2 is a
gas at the start, pressure is 300 psia and Dh = 160 Btu/lb
Basis: 10 lb CO2
In the energy balance: DE = Q + W
W = 0 (volume of the system is fixed, DK =DP = 0), Q = DU
= Dh – D(pv)
Find Dhfinal from the CO2 chart by following the constant-
volume line of 0.4 ft3/lb to the spot where the quality is
0.6.
Dhfinal = 81 Btu/lb, pfinal = 140 psia
Q = (81-160)-{[(140)(144)(0.4)/778.2] –
[(300)(144)(0.4)/778.2]} = -67.2 Btu/lb (heat is removed)

Example 3.10
Argon gas in an insulated plasma deposition
chamber with a volume of 2 L is to be heated by
an electric resistance heater. Initially the gas,
which can be treated as an ideal gas, is at 1.5 Pa
and 300 K. The 1000-ohm heater draws current
at 40 V for 5 minutes (480 J of work is done by
the surrounding). What is the final gas
temperature and pressure at equilibrium? The
mass of the heater is 12 g and its heat capacity is
0.35 J/gK. Assume that the heat transfer to the
chamber from the gas at this low pressure and in
the short time period is negligible.

By Assist. Prof. Dr. Wipada Sanongraj 111


1304 211 Chemical Engineering Principles
and Calculations

Solution
Closed system: DK=DP = 0
DE=Q + W = DU
Q = 0 so DE = W = DU
Basis: 5 minutes
Calculate mass of the gas: n = pV/RT
n = 1.5 Pa 2L 10-3 m3 1 gmol K
1L 8.314 Pa m3 300 K
= 1.203*10-6 gmol
Cv = Cp – R and since Cp = 5/2R, so Cv = 3/2R
DU = nCv(T-300)
480 J = (12g)(0.35 J/g K)(T-300) + (1.203*10-6) (3/2)(8.214)(T-
300)
T = 414.3 K
Final pressure is p2 = p1(T2/T1) = 1.5(414.3/300) = 2.07 Pa

Example 3.11
Air is being compressed from 100 kPa and 255 K
(where it has an enthalpy of 489 kJ/kg) to 1000
kPa and 278 K (where it has an enthalpy of 509
kJ/kg). The exit velocity of the air from the
compressor is 60 m/s. What is the power
required (in kW) for the compressor if the load is
100 kg/hr of air?
225 K 278 K
Dh = 489 kJ/kg Dh = 509 kJ/kg
v1 = 0 v1 = 60
100 kPa 1000 kPa

By Assist. Prof. Dr. Wipada Sanongraj 112


1304 211 Chemical Engineering Principles
and Calculations

Solution
Basis: 100 kg of air = 1 hr
Simplify energy balance: DE = Q + W – D[(h+k+p)m]
The process is in steady state: DE = 0
m1=m2 = m
D(pm) = 0
Q = 0 by assumption
W = DH + DK
DH = (509-489)kJ/kg*100 kg = 2000 kJ
DK = 1/2m(v22-v12) = ½(100 kg)(602 m2/s2) = 180 kJ
W = 2180 kJ
Convert to power
kW = 2180 kJ/1 hr (=3600 sec) = 0.61 kW

Example 3.12
Water is being pumped from the bottom of a well 15 feet
deep at the rate of 200 gal/hr into a vented storage tank
to maintain a level of water in a tank 165 ft above the
ground. To prevent freezing in the winter a small heater
puts 30,000 Btu/hr into the water during its transfer
from the well to the storage tank. Heat is lost from the
whole system at the constant rate of 25,000 Btu/hr. What
is the temperature of the water as it enters the storage
tank, assuming that the well water is at 35 F? A 2-hp
pump is being used to pump the water. About 55% of
the rated horsepower goes into the work of pumping
and the rest is dissipated as heat to the atmosphere.

By Assist. Prof. Dr. Wipada Sanongraj 113


1304 211 Chemical Engineering Principles
and Calculations

Solution
Open system with flow in and out
Material balance is 200 gal enter and 200 gal leave in an
hour
Energy balance is : DE = Q + W – D(h+k+p)m
Process is in the steady state, DE = 0
m2 = m1 = m
DK = 0 because will assume that v1 = v2 = 0, then 0 = Q + W
– D(h + p)m
At the top of the tank, DH = mDh = mCp(T2-35)
Total mass of water pumped is (200 gal/hr)(8.33 lb/1gal) =
1666 lb/hr
Potential energy change = (1666 lb/hr)(32.2
ft/s2)(180ft)/(32.2 ft lbm/s2 lbf)/(778 ft lbf) = 385.5 Btu

Heat lost by system is 25,000 Btu while the heater puts


30,000 Btu into the system, hence the net heat exchange =
Q = 30000 – 25000 = 5000 Btu
Rate of work being done on the water by the pump = w =
(2 hp)(0.55)(33,000 ft lbf/min hp)(60 min/hr)(1 Btu/778
ft lbf) = 2800 Btu/hr
DH can be calculated from: Q + W = DH + DP = 7414 Btu
Heat capacity of liquid water assumed to be constant and
equal to 1.0 Btu/lb F
7414 = DH = 1666(1.0)DT
DT = 4.5 F, T = 39.5 F

By Assist. Prof. Dr. Wipada Sanongraj 114


1304 211 Chemical Engineering Principles
and Calculations

Energy Balance with


Reaction
Standard heat of formation
For single species and mixture, specific
enthalpy change from the reference state

T
DHA = DH ofA + òC
Tref
dT
pA

s s T
DHmixture = åni DH of i + å ò nC
i pi dT
i =1 i =1 Tref

Heat of formation Sensible heat

For example, suppose we have species 1 and 2


enter system, react, and species 3 and 4 leave.
Then

DHout -DHin = (n3DHof 3 + n4DHof 4 ) - (n1DHof 1 + n2DHof 2 )


Tout Tin
+ ò (n3Cp3 + n4Cp4 )dT - ò (nC1 p1 - n2Cp2 )
Tref Tref

±Enthalpies associated with phase change

DH rxn = ( å
products
ni DH f i - å n DH
reac tan ts
i fi )

By Assist. Prof. Dr. Wipada Sanongraj 115


1304 211 Chemical Engineering Principles
and Calculations

Example 3.13
Calculate DHrxno for the following reaction of 4 g mol of NH3
4NH3(g) + 5O2(g) 4NO(g) + 6H2O(g)

Basis: 4 g mol of NH3


Data NH3(g) O2(g) NO(g) H2O(g)
DHfo per mole-46.191 0 +90.374 -241.826
At 25 C 1atm
(kJ/g mol)
DHrxno = [4(90.374)+6(-241.826)]-[5(0) + 4(-46.191)]

= -904.696 kJ/4 g mol NH3

Example 3.14
If the standard heat of formation for H2O(l)
is -285.838 kJ/g mol and the heat of
evaporation is 44.012 kJ/g mol at 25 OC
and 1 atm, what is the standard heat of
formation of H2O(g)?

By Assist. Prof. Dr. Wipada Sanongraj 116


1304 211 Chemical Engineering Principles
and Calculations

Solution
DHrxn=SDHf,product-SDHf,reactants
A: H2(g) + 1/2O2(g) H2O(l)
DHrxno=-285.838 kJ/g mol
B: H2O(l) H2O(g)
DHvap = 44.012 kJ/g mol
A+B: H2(g) + 1/2O2(g) H2O(l)
DHrxn + DHvap = DHrxn H2O(g) = DHfoH2O(g)
o o

= - 241.826 kJ/ gmol

Example 3.15
An iron pyrite containing 85% FeS2 and 15%
gangue (inert dirt, rock) is roasted with an
amount of air equal to 200% excess air according
to the reaction
4FeS2 + 11O2 2Fe2O3 + 8SO2

In order to produce SO2. All the gangue plus the


Fe2O3 end up in the solid waste product (cinder),
which analyzes 4% FeS2. Determine the heat
transfer per kg of ore to keep the product stream
at 25 oC if the entering steams are at 25 oC

By Assist. Prof. Dr. Wipada Sanongraj 117


1304 211 Chemical Engineering Principles
and Calculations

Solution
Basis: 100 kg of pyrite ore
MW of Fe(55.85), Fe2O3(159.7), FeS2(120)

Product:P
Mol%
SO2 x4
O2 x5
200% xs air N2 x6 Cinder:C
Wt%
Mol%
Gangue x1
N2 79
Fe2O3 x2
O2 21
FeS2 x3
Total 100
Total x1+x2+x3
Ore:F
Wt%
FeS2 85
Gangue 15

The excess air is: Mol FeS2 = (85/120) = 0.7083 kg mol


Required O2 = 0.7083(11/4) = 1.9479 kg mol
Excess O2 = 1.9479(2.0) = 3.8958
Total O2 in = 5.8437 kg mol
Total N2 in = 5.8437(79/21) = 21.983 kg mol
Element mass balance are
In Out
Gangue 15 x1
N2(kg mol) 21.983 x6
S (kg mol) 2(85/120) x4+ (x3/120)(2)
Fe (kg mol) 1(85/120) (x2/159)2 + (x3/120)(1)
O2 (kg mol) 5.8437 x4 + x5 + (x2/159.7)(1.5)

We know x3/(x1+x2+x3) = 0.04

By Assist. Prof. Dr. Wipada Sanongraj 118


1304 211 Chemical Engineering Principles
and Calculations

The solution of these equation is


In P In C
SO2 = 1.368 kg mol Gangue = 15 kg
O2 = 3.938 Fe2O3 = 54.63 kg = 0.342 kg mol
N2 = 21.983 FeS2 = 2.9 kg = 0.0242 kg mol

General energy balance reduce to DE = 0, DP = 0, DK = 0, W = 0,


so Q = DH. Choose reference state to be 25 oC and 1 atm with
the result that all the sensible heat terms become zero
Q = SniDHio - SniDHio
Products reactants

Comp 10-3 g mol DHfo (kJ/g niDH fi o(kJ) 10-3 g DHfo (kJ/g niDH fio
mol) mol mol) (kJ)
FeS2 0.0242 -177.9 -4.305 0.7083 -177.9 -126.007
Fe2O3 0.342 -822.156 -281.177 0 -822.156 0
N2 21.9983 0 0 21.983 0 0
O2 3.938 0 0 5.8437 0 0
SO2 1.368 -296.9 -406.159 0 -296.9 0
-691.641 -126.007

Q = [-691.641-(-126.007)](103) = -565.634*103 kJ/100 kg ore


Q = -5.656*103 kJ/kg ore

By Assist. Prof. Dr. Wipada Sanongraj 119


1304 211 Chemical Engineering Principles
and Calculations

Chapter 4 : Gases, Vapors,


Liquids, and Solids

Gases, Vapors, Liquids, and Solids


The Ideal Gas Law
pV = nRT

Where p = absolute pressure of the gas


V = Total volume occupied by the gas
n = number of moles of the gas
R = ideal gas constant in appropriate units
T = absolute temperature of the gas
Sometimes the ideal gas law is written as:

pVˆ = RT
Vˆ = specific volume of the gas (vol. per mole)

By Assist. Prof. Dr. Wipada Sanongraj 120


1304 211 Chemical Engineering Principles
and Calculations

Phase-rule variables are variables of the kind with


which the phase rule is concerned and they are
intensive properties of the system.
Intensive properties are the ones that do not
depend on the quantity of the material present. The
specific (per unit mass) values are intensive
properties.

Extensive properties are the ones that depend on


how much material we have. The total quantities
are extensive properties.

An example of the use of the phase rule is the ideal


gas law, PV = nRT. In order to be able to determine
the remaining one unknown, you might conclude
that F = 3. However, if we apply the phase rule for
a single phase P = 1 and for a pure gas C = 1, so
that
F=C–P+2=1–1+2=2
How can we explain this apparent paradox with our
previous statement? Since the phase rule is
concerned with intensive properties only, the
following are phase-rule variables in the ideal gas
law:
P, T, and Vˆ

By Assist. Prof. Dr. Wipada Sanongraj 121


1304 211 Chemical Engineering Principles
and Calculations

Thus the ideal gas law would be written as

pVˆ = RT

We can see that by specifying two intensive variables


(F = 2), the third can be calculated.

Example 4.1 Application of the phase rule


Calculate the number of degrees of freedom from
the phase rule for the following materials at
equilibrium:

By Assist. Prof. Dr. Wipada Sanongraj 122


1304 211 Chemical Engineering Principles
and Calculations

In many processes going from an initial state to a


final state, you can use the ratio of the ideal gas
laws in the respective states and eliminate R as
follows:

p 1V 1 n RT
= 1 1
p 2V 2 n 2 RT 2

or
æ p1 öæ V1 ö æ n1 öæ T1 ö
ç ÷ç ÷ = ç ÷ç ÷
çp ÷çè V ÷ø çè n ÷øçè T ÷ø
è 2 ø 2 2 2

Example 4.2: Use of Standard Conditions


Calculate the volume, in cubic meters, occupied by
40 kg of CO2 at standard conditions.

Solution : Basis: 40 kg of CO2

40 kg CO2 1 kgmol CO2 22.4 m3 CO2 = 20.4 m3 CO2 at S.C.


44 kg CO2 1 kgmol CO2

By Assist. Prof. Dr. Wipada Sanongraj 123


1304 211 Chemical Engineering Principles
and Calculations

Example 4.3 : Calculation of Gas Density

What is the density N2 at 27 oC and 100 kPa in SI units?

Solution : Basis: 1 m3 of N2 at 27 oC and 100 kPa

1 m3 273 K 100 kPa 1kgmol 28 kg = 1.123 kg


300 K 101.3 kPa 22.4 m3 1kgmol

density = 1.123 kg/m3 of N2 at 27 oC and 100 kPa

The specific gravity of a gas is usually defined as


the ratio of the density of the gas at a desired
temperature and pressure to that of air at a certain
temperature and pressure.
Example 4.4 : Specific gravity of a gas
What is the specific gravity of N2 at 80 oF and 745
mm Hg compared to air at 80 oF and 745 mm Hg?
Basis : 1 ft3 of air at 80 oF and 745 mm Hg
1 ft3 492 oR 745 mmHg 1lbmol 29 lb = 0.0721 lb/ft3
540 oR 760 mm Hg 359 ft3 1 lbmol

By Assist. Prof. Dr. Wipada Sanongraj 124


1304 211 Chemical Engineering Principles
and Calculations

Basis : 1 ft3 of N2 at 80 oF and 745 mm Hg

1 ft3 492 oR 745 mmHg 1lbmol 28 lb = 0.0697 lb/ft3


540 oR 760 mm Hg 359 ft3 1 lbmol

lbN 2 / ft 3 at 80 o F , 745 mmHg


(sp.gr.)N 2 = 0.0697
= 0.967
0.0721 lbair / ft 3 air at 80 o F ,745 mmHg

Ideal Gas Mixtures and Partial Pressure


The partial pressure of gas i defined by Dalton, pi,
namely the pressure that would be exerted by a
single component in a gaseous mixture if it existed
by itself in the same volume as occupied by the
mixture and at the same temperature of the mixture
is
piVtotal = niRTtotal

piVtotal n RT
= i total
or ptotalVtotal ntotal RTtotal
ni
p i = p total = p total y i
n total
yi is the mole fraction of component i

By Assist. Prof. Dr. Wipada Sanongraj 125


1304 211 Chemical Engineering Principles
and Calculations

Example 4.5 : Calculation of Partial Pressures


from a Gas Analysis
A flue gas analyzes 14% of CO2, 6% of O2, and
80% of N2. It is a t 400 oF and 765 mmHg pressure.
Calculate the partial pressure of each component.
Use pi = ptyi

Basis : 1.00 kg (or lb) mol of flue gas


Component kg (or lb) mol p (mm Hg)
CO2 0.140 107.1
O2 0.060 45.9
N2 0.800 612.0
Total 1.000 765.0

Material Balances Involving gases


Example 4.6 : Material Balance with Combustion

A gas produced by gasifying wood chips analyzes


6.4 % CO2, 0.1% O2, 39%CO, 51.8% He, 0.6%CH4,
and 2.1% N2. It enters the combustion chamber at
90 oF and a pressure of 35.0 in Hg. And is burned
with 40% excess air (dry) which is at 70 oF and the
atmospheric pressure of 29.4 in Hg, 10% of the CO
remains unburned. How many cubic feet of air are
supplied per cubic foot of entering gas? How many
cubic feet of product are produced per cubic foot of
entering gas if the exit gas is at 29.4 in Hg and 400
oF?

By Assist. Prof. Dr. Wipada Sanongraj 126


1304 211 Chemical Engineering Principles
and Calculations

90 oF and 35.0 in Hg 400 oF and 29.4 in Hg


Gas 100 lb mol Product
Combustion
Comp. % = mol O2 req. P lbmol
CO2 6.4 - Air A = ? CO2 ?
O2 0.1 (0.1) lb mol H2O ?
CO 39.0 19.5 O2 0.21 CO ?
H2 51.8 25.9 N2 0.79 O2 ?
CH4 0.6 1.2 1.00 N2 ?
40% xs
N2 2.1 -
70 oF and 29.4 in Hg
100.0 46.5

Basis : 100 lbmol synthesis gas, % = lbmol

The entering air can be calculated from the


specified 40% excess air, the reactions for
complete combustion are

1
CO + O 2 ® CO 2
2
1
H 2 + O2 ® H 2 O
2
CH4 + 2O2 ®CO2 + 2H2O
The excess is
Excess O2 : 0.4(46.5) = 18.6 lb mol
Total O2 : 46.5 + 18.6 = 65.10 lb mol

By Assist. Prof. Dr. Wipada Sanongraj 127


1304 211 Chemical Engineering Principles
and Calculations

N2 in is 65.10(79/21) = 244.9 lb mol


Total moles of air in are 244.9 + 65.10 = 310 lb mol.
Element balances in moles
In Out
N2: 2.1 + 244.9 = nN2
H2: 51.8 + 0.6(2) = nH2O
C: 6.4+ 39.0+0.6 = nCO2 + 0.10(39.0)
O2: 6.4+0.1+0.5(39)+65.1 = nO2 + nCO2 + 0.5(nH2O
+nCO)
From 10% of CO remains unburned so nCO = 3.9%

The solutions of these equations is

nN2 = 247.0 nCO = 3.9 nCO2 = 42.1 nH2O = 53.0


nO2 = 20.55
The total moles exiting sum up to be 366.55 lbmol.

Then we will convert the lbmol of air and product


into the volumes requested:
Tgas = 90 + 460 = 550 oR
Tair = 70 + 460 = 530 oR
Tproduct = 400 + 460 = 860 oR

By Assist. Prof. Dr. Wipada Sanongraj 128


1304 211 Chemical Engineering Principles
and Calculations

ft 3 of gas
100 lbmol entering gas (359 ft 3 at S .C.)(550 o R)(29.92 in.Hg )
=
1 lbmol (492 o R)(35.0 in. Hg )
= 343 *102
ft 3 of air
310 lbmol air (359 ft 3 at S .C.)(530 o R)(29.92 in.Hg )
=
1 lbmol (492 o R)(29.4 in. Hg )
= 1220 *102

ft 3 of product
366.55 lbmol P (359 ft 3 at S .C.)(860 o R)(29.92 in.Hg )
=
1 lbmol (492 o R)(29.4 in. Hg )
= 2340 *102

The answers to the questions are

1220 * 10 2 ft 3 air at 530 o R and 29.4 in. Hg


= 3.56 3
343 * 10 2 ft gas at 550 o R and 35.0 in. Hg

2340 *102 ft 3 product at 860 o R and 29.4 in. Hg


= 6.82
343 *102 ft 3 gas at 550 o R and 35.0 in. Hg

By Assist. Prof. Dr. Wipada Sanongraj 129


1304 211 Chemical Engineering Principles
and Calculations

Critical State, Reduced Parameters, and Compressibility

The law of corresponding states expresses the idea


that in the critical state all substances should
behave alike.

The critical state for the gas-liquid transition is the


set of physical conditions at which the density and
other properties of the liquid and vapor become
identical.

This point, for a pure component (only), is the highest


temperature at which liquid and vapor can exist in
equilibrium.

A supercritical fluid, a compound in a state above the


critical point, combines some of the properties of both
gases and liquids. Supercritical fluids are used to
replace the void left by solvents such as
trichloroethylene and methylene chloride.

Figure 1 The region of existence of solid, liquid, gaseous, and supercritical


water. At the triple point solid, liquid, and gas are all in equilibrium
(Himmelblau, 1996)

By Assist. Prof. Dr. Wipada Sanongraj 130


1304 211 Chemical Engineering Principles
and Calculations

Reduced parameters are corrected or normalized


conditions of temperature, pressure, and volume,
normalized by their respective critical conditions.

Tr = T/Tc

Pr = P/Pc


V r
=
Vˆ c

The common way to modify the ideal gas law to a


real gas law is to insert an adjustable coefficient, z,
the compressibility factor in it. The compressibility
factor is a factor that compensates for the
nonideality of the gas. Thus the ideal gas law
becomes a real gas law, a generalized equation of
state.
If the compressibility factor is plotted for a given
temperature against the pressure for different
gases, we obtain plots as shown in Figure 2.
However, if the compressibility is plotted against the
reduced pressure as a function of the reduced
temperature, then for most gases the
compressibility values at the same reduced temp.
and pressure fall at about the same point.

By Assist. Prof. Dr. Wipada Sanongraj 131


1304 211 Chemical Engineering Principles
and Calculations

Figure 2 (a) Compressibility factor as a function of temperature and pressure


(b) compressibility as a function of reduced temperature and reduced
pressure (Himmelblau, 1996).

Figure 3 (a) Generalized compressibility chart showing the


respective portions of the subsequent expanded charts
(Himmelblau, 1996)

By Assist. Prof. Dr. Wipada Sanongraj 132


1304 211 Chemical Engineering Principles
and Calculations

Figure 3 (b) Generalized compressibility chart, very low


reduced pressure (Himmelblau, 1996).

Figure 3 (c) Generalized compressibility chart, low pressure


(Himmelblau, 1996).

By Assist. Prof. Dr. Wipada Sanongraj 133


1304 211 Chemical Engineering Principles
and Calculations

Figure 3 (d) Generalized compressibility chart, medium


pressure (Himmelblau, 1996).

Figure 3 (e) Generalized compressibility chart, high


pressure (Himmelblau, 1996).

By Assist. Prof. Dr. Wipada Sanongraj 134


1304 211 Chemical Engineering Principles
and Calculations

Figure 3 (f)

Figure 3 shows the generalized compressibility charts


or z-factor chart prepared by Nelson and Obert. These
charts are based on 30 gases. Figure 3 (b) and (c)
represent z for 26 gases (excluding H2, He, NH3, H2O)
with a maximum deviation of 1%, and H2 and H2O within
a deviation of 1.5%. Figure 3 (d) is for 26 gases and is
accurate to 2.5%, while Figure 3 (e) is for 9 gases and
errors can be as high as 5%. For H2 and He only,
corrections to the actual critical constants are used to
give pseudocritical constants which enable us to use
Figure 3 (a-e) for these two gases as well with minimum
error.

Tc' = Tc + 8 K

Pc' = Pc + 8 atm

By Assist. Prof. Dr. Wipada Sanongraj 135


1304 211 Chemical Engineering Principles
and Calculations

Figure 3 (f) is a unique chart that, by having several


parameters plotted simultaneously on it, helps us
avoid trial-and-error solutions or graphical solutions
of real gas problems. One of these helpful
parameters is the ideal reduced volume defined as:


V ri =
Vˆci
Where Vci is the ideal critical volume (not the
experimental value of the critical volume), or
RT
V ci =
c

Pc

All we need to know to use these charts are the


critical temperature and the critical pressure for a
pure substance. The value of z = 1 represents
ideality and the value z = 0.27 is the compressibility
factor at the critical point.

By Assist. Prof. Dr. Wipada Sanongraj 136


1304 211 Chemical Engineering Principles
and Calculations

Example 4.7 : Use of the compressibility factor


In spreading liquid ammonia fertilizer, the charges
for the amount of NH3 are based on the time
involved plus the pounds of NH3 injected into the
soil. After the liquid has been spread, there is still
some ammonia left in the source tank (volume = 120
ft3 ), but in the form of a gas. Suppose that your
weight tally, which is obtained by difference, shows a
net weight of 125 lb of NH3 left in the tank at 292
psig. Because the tank is sitting in the sun, the
temperature in the tank is 125 oF. Your boss
complains that his calculations sow that the specific
volume of the gas is 1.2 ft3/lb, and hence that there
are only 100 lb of NH3 in the tank. Could he be
correct?

Basis: 1 lb of NH3
Apparently, your boss used the ideal gas law in
getting his figure of 1.2 ft3/lb of NH3 gas.
( psia)( ft 3 )
R = 10.73 P = 292 + 14.7 = 306.7 psia
(lbmol )( o R)

1lb
T = 125 oF + 460 = 585 oR n=
17lb / lbmol
1
(10 . 73 )( 585 )
RT 17
V =
ˆ = = 1 . 20 ft 3 / lb
P 306 . 7

By Assist. Prof. Dr. Wipada Sanongraj 137


1304 211 Chemical Engineering Principles
and Calculations

However, he should have used the compressibility


factor, because NH3 does not behave as an ideal
gas under the observed conditions of temperature
and pressure. Let us again compute the mass of
gas in the tank this time using
PV = znRT

Tc = 405.5 K = 729.9 R
Pc = 111.3 atm = 1636 psia

Then, since z is the function of Tr and Pr

T 585 o R
Tr = = = 0 . 801 Pr =
P
=
306 . 7 psia
= 0 . 187
T c 729 . 9 o R Pc 1636 psia

From the Nelson and Obert chart, Figure 3 (c), we


can read z = 0.855.

Vˆ = 1 . 2 ( ft 3 / lb ) ideal ( 0 .855 )
= 1 .03 ft 3 / lbNH 3

Mass of NH3 = (120 ft3)(1lb NH3/1.03 ft3)

By Assist. Prof. Dr. Wipada Sanongraj 138


1304 211 Chemical Engineering Principles
and Calculations

Equation of State
Another way to predict p, V, n, and T for real gases

Equations of state are formulated by collecting


experimental data and calculating the coefficients
in a proposed equation by statistical fitting.
Van der Waals is the first one who generated
equation of state considering 2 effects that make
real gas behaves different from ideal gas.
1)Interaction between molecules called Van der
Waals force causing the reduction of pressure.
2) Volume of molecule is not negligible as
compared to volume of gas.

Van der Waals’ Equation can be expressed as:

nRT n 2a
p = -
V - nb V 2

However, if you want to solve for V (or n), you can see
that the equation becomes cubic in V (or n):

æ nRT ö 2 n 2 a n 3 ab
f (V ) = V 3 - ç nb + ÷ V + V - =0
è p ø p p

when a and b are the van der Waals constants obtained


from experiments.

By Assist. Prof. Dr. Wipada Sanongraj 139


1304 211 Chemical Engineering Principles
and Calculations

Example 4.8: Application of Van der Waals’ Equation


A cylinder 0.15 m3 in volume containing 22.7 kg of
propane C3H8 stands in the hot sun. A pressure
gauge shows that the pressure is 4790 kPa gauge.
What is the temperature of the propane in the
cylinder? Use van der Waals equation.
Basis : 22.7 kg of propane
The van der Waals constants are
a = 9.24*106 atm(cm3/gmol)2 b = 90.7 cm3/gmol
n 2a
p + (V - n b ) = n R T
V 2

All the additional information you need is as follows:


(4790 + 101) kPa ´ 1 atm
P= = 48.3 atm abs
101.3 kPa

( c m 3 )( a tm )
R = 8 2 .0 6
( g m o l )( K )

22.7 kg
n= = 0.516 kgmol propane
44 kg / kgmol

é ( 0 .5 1 6 ´ 1 0 3 ) 2 ( 9 .2 4 ´ 1 0 6 ) ù
ê 4 8 .3 + ú [ 0 .1 5 0 ´ 1 0
6

ë ( 0 .1 5 ´ 1 0 6 ) 2 û
- ( 0 .5 1 6 ´ 1 0 3 ) ( 9 0 .7 ) ] = ( 0 .5 1 6 ´ 1 0 3 ) (8 2 .0 6 ) ( T K )

T = 384 K

By Assist. Prof. Dr. Wipada Sanongraj 140


1304 211 Chemical Engineering Principles
and Calculations

Example 4.9: Solution of Van der Waals’ Equation for V


Given the values of
P = 679.7 psia a = 3.49*104 psia(ft3/lbmol)2

n = 1.136 lbmol R = 10.73 (psia)(ft3)/(lbmol)(oR)


T = 683 oR b = 1.45 (ft3/lbmol)
Solve for the volume of the vessel
Write van der Waals’ equation as a cubic equation
in one unknown variable, V.

æ pnb + nRT ö 2 n 2 a n 3 ab
f (V ) = V 3 - ç ÷V + V- =0
è p ø p p

Let us apply Newton’s method to obtain the desired


root:
f (V k ) (a)
V =V -
k +1 k
f '(V k )

Where f’(Vk) is the derivative of f(V) with respect to V


evaluated at Vk:
æ pnb + nRT ö n2a
f '(V ) = 3V 2 - 2 ç ÷ V +
è p ø p

You can obtain a reasonably close approximation to


V (or n) in many cases from the ideal gas law, useful
at least for the first trial in which k = 0 in Eq. (a)

By Assist. Prof. Dr. Wipada Sanongraj 141


1304 211 Chemical Engineering Principles
and Calculations

nRT 10.73( psia)( ft 3 ) 1


Vo = = 1.136lbmol ´ o
´ 683o R ´
P (lbmol)( R) 679.7 psia

= 12.26 ft3 at 679.7 psia and 683 oR

The second and subsequent estimates of V will be


calculated using Eq.(a):
f (V o )
V1 = V o -
f '( V o )

(679.7)(1.137)(1.45) + (1.137)(10.73)(683)
f (Vo ) = (12.26)3 - (12.26)2
679.7
(1.137)2 (3.49 ´104 ) (1.137)3 (3.49 ´104 )(1.45)
+ (12.26) - = 738.3
679.7 679.7

2[(679.7)(1.137)(1.45) + (1.137)(10.73)(683)]
f '(Vo ) = 3(12.26)2 - (12.26)
679.7
(1.137)2 (3.49 ´104 )
+ = 216.7
679.7

V1 = 12.26 - 738.3/216.7 = 8.85


On the next iteration
f (V 1 )
V 2 = V1 -
f '(V 1 )

The final solution is 5.0 ft3 at 679.7 psia and 683 oR.

By Assist. Prof. Dr. Wipada Sanongraj 142


1304 211 Chemical Engineering Principles
and Calculations

Gaseous Mixtures

Use Kay’s method of pseudocritical values to


calculate the pseudo-reduced values and predict
P, V, T, and n via the compressibility factor.

In Kay’s method, pseudocritical values for mixtures


of gases are calculated on the assumption that
each component in the mixture contributes to the
pseudocritical value in the same proportion as the
number of moles of that component. Thus, the
pseudocritical values are computed as follows:

Pc' = PcA y A + PcB y B + ...

Tc' = TcA y A + TcB y B + ...

Where yi is the mole fraction


Pc’ is the pseudocritical pressure
Tc’ is the pseudocritical temperature

The pseudoreduced variables can be calculated


as follows:
P T
P = '
'
Tr' =
r
Pc Tc'

By Assist. Prof. Dr. Wipada Sanongraj 143


1304 211 Chemical Engineering Principles
and Calculations

Example 4.10: Calculation of P-V-T Properties for


Real Gas Mixture

A gaseous mixture has the following composition (in


mole percent)

Methane 20
Ethylene 30
Nitrogen 50

at 90 atm pressure and 100 oC. Compare the volume


per mole as computed by the method of:

a)The perfect gas law


b) the pseudoreduced technique (Kay’s method)

Basis : 1 gmol of gas mixture


Additional data needed are:
Component Tc (K) Pc (atm)
Methane 191 45.8
Ethylene 283 50.9
Nitrogen 126 33.5

R = 82.06 (cm3-atm)/(gmol-K)

By Assist. Prof. Dr. Wipada Sanongraj 144


1304 211 Chemical Engineering Principles
and Calculations

a) Perfect gas law:


nRT 1(82.06)(373)
Vˆ = =
P 90
= 340 cm 3 at 90 atm and 373 K
b) According to Kay’s method, we first calculate the
pseudocritical values for the mixture

Pc' = PcA y A + PcB y B + PcC yC


= ( 45.8)(0.2) + (50.9)(0.3) + (33.5)(0.5)
= 41.2 atm

Tc' = TcA y A + TcB y B + TcC yC


= (191)(0.2) + (283)(0.3) + (126)(0.5)
= 186 K
Then we calculate the pseudoreduced values for the
mixture

P 90
Pr' = = = 2.18
Pc' 41.2

T 373
Tr' = '
= = 2.01
Tc 186

By Assist. Prof. Dr. Wipada Sanongraj 145


1304 211 Chemical Engineering Principles
and Calculations

By using these two parameters and the


generalized compressibility chart, we will get z =
0.965. Thus

znRT 0.965(1)(82.06)(373)
Vˆ = =
P 90
= 328 cm 3 at 90 atm and 373 K

Vapor Pressure and Liquids

Figure 4 Vapor pressure curve for water


(Himmelblau, 1996).

By Assist. Prof. Dr. Wipada Sanongraj 146


1304 211 Chemical Engineering Principles
and Calculations

Change of Vapor Pressure with Temperature

Because the function of vapor pressure (P) vs. T is


not a linear function so we use the Antoine equation
to predict P from T
B
ln( P*) = A -
C +T
Where A, B, C = constants for each substance
T = temperature, K

Example 4.11 : Vaporization of Metals for Thin Film


Deposition
The following figure shows evaporation from a boat
placed in a vacuum chamber. The boat made of
tungsten has a negligible vapor pressure at 972 oC,
the operating temperature for the vaporization of Al
(which melts at 660 oC and fills the boat). The
approximate rate of evaporation is given in g/(m2)s
by * 1/ 2
P ( MW )
m = 0.437
T 1/ 2

Where P* is the pressure in kPa and T is the


temperature in K.
What is the vaporization rate for Al at 972 oC in
g/(cm2)s?

By Assist. Prof. Dr. Wipada Sanongraj 147


1304 211 Chemical Engineering Principles
and Calculations

Target
Vacuum
chamber

To pump
electrode electrode

Solution
We have to calculate P* for Al at 972 oC. The Antoine
equation is suitable if data are known for the vapor
pressure of Al. Considerable variation exists in the
data for Al at high temperature, but we will use A =
8.779, B = 1.615*104, and C = 0 with P* in mm Hg
and T in K.
1.615*104
*
ln P972 = 8.799 -
972 + 273
o
C

= 0.0154 mmHg (0.00201 kPa)


1
2
( 0 .0 0 2 0 1) ( 2 6 .9 8 )
m = 0 .4 3 7 1
(9 7 2 + 2 7 3 ) 2

-4
= 1 .3 * 1 0 g /(c m 2
)( s )

By Assist. Prof. Dr. Wipada Sanongraj 148


1304 211 Chemical Engineering Principles
and Calculations

Another way to relate vapor pressure to temperature


is by a graphical technique. The curvature for P* vs.
T can be straightened out by a special plot known as
a Cox chart.

How to make the Cox chart


1. Mark on the horizontal scale values of logP* as to
cover the desired range of P*
2. Next draw a straight line on the plot at a suitable
angle, say 45o, that covers the range of P*
3. To calibrate the vertical axis in common integers
such 25, 50, 100, 200 degrees, and so on, you
use a reference substance, namely water. For the
first integer, say 100 oF, you look up the vapor
pressure of water in the steam tables, or calculate
it from the Antoine equation, to get 0.9487 psia.
Locate this value on the horizontal axis, and
proceed vertically until you hit the straight line.
Then proceed horizontally until you hit the vertical
axis. Mark the scale there as 100 oF.

By Assist. Prof. Dr. Wipada Sanongraj 149


1304 211 Chemical Engineering Principles
and Calculations

4. Pick the next temperature, say 200 oF.


5. Continue as in 3 and 4 until the vertical scale is
established over the desired range for the
temperature.

Figure 5 Cox chart (Himmelblau, 1996).

By Assist. Prof. Dr. Wipada Sanongraj 150


1304 211 Chemical Engineering Principles
and Calculations

Saturation
Saturate : method, ew first calculate the .779, B = 1.615*10
E 1996).•n volume containing .2 7 kg of
Saturated Air : RaaodDr o

When any pure gas (or a gaseous mixture) comes in


contact with a liquid, the gas will acquire molecules
from the liquid. If contact is maintained for a
considerable length of time, vaporization continues
until equilibrium is attained (no more liquid will
vaporize into the gas phase), the gas is then said to
be saturated with the particular vapor at the given
temperature.

PairV nairRT
=
PH2OV nH2O RT

Figure 6 Evaporation of water at constant


pressure and temperature of 65 oC

By Assist. Prof. Dr. Wipada Sanongraj 151


1304 211 Chemical Engineering Principles
and Calculations

Figure 7 Change of partial and total pressures during


the vaporization of water into air at constant
temperature.

Dew point for the mixture of pure vapor and


noncondensable gas means the temperature at
which the vapor just starts to condense when
cooled at constant pressure.
The dew point is the temperature you must cool
air at constant pressure in order for that air mass
to become saturated. This does not necessarily
mean a cloud or rain drop will form. When water
does condense into a cloud or rain drop, it means
that the air temperature and dew point
temperature are the same, and this would mean
we have 100% relative humidity. High dew points
mean high moisture content of the air, which
often translates to muggy and uncomfortable
conditions.

By Assist. Prof. Dr. Wipada Sanongraj 152


1304 211 Chemical Engineering Principles
and Calculations

The most common confusion over the dew point


is that it is the temperature at which condensation
forms. This is not necessarily true.
Actually condensation is always occurring in our
air. At the dewpoint temperature however, is
when condensation overtakes evaporation, and
this is a step in the process of forming dew,
clouds, rain, fog, basically water droplets.

Example 4.12 : Saturation


What is the minimum number of cubic meters of
dry air at 20 oC and 100 kPa necessary to
evaporate 6.0 kg of ethyl alcohol if the total
pressure remains constant at 100 kPa and the
temperature remains 20 oC? Assume that the air is
blown through the alcohol to evaporate it in such a
way that the exit pressure of the air-alcohol mixture
is at 100 kPa.

Solution
Assuming that the process is isothermal

By Assist. Prof. Dr. Wipada Sanongraj 153


1304 211 Chemical Engineering Principles
and Calculations

The additional data needed are

MW. of ethyl alcohol (C2H5OH) = 46.07


P*alcohol
B
From ln( P*) = A -
C +T

A = 18.5242 B = 3578.91 C = -50.50

P*alcohol = 43.647 mm Hg (5.82 kPa)

We will use P*alcohol = 5.93 kPa

20 oC 100 kPa
100 kPa Saturated
Air air-alcohol
mixture

Alcohol
6 kg

Basis : 6.0 kg of alcohol

By Assist. Prof. Dr. Wipada Sanongraj 154


1304 211 Chemical Engineering Principles
and Calculations

The ratio of moles of ethyl alcohol to moles of air


in the final gaseous mixture is the same as the
ratio of the partial pressures of these two
substances. Since we know the moles of alcohol,
we can find the number of moles of air.

*
Palcohol n
= alcohol
Pair nair

And once we know the number of moles of air we


can apply the ideal gas law.

Pair = Ptotal – P*alcohol = (100-5.93) kPa = 94.07 kPa

6 kg alcohol ´ 1 kgmol alcohol ´ 94.07 kgmol air


nair =
46.07 kg alcohol ´ 5.93 kgmol alcohol
= 2.07 kgmol air
Using Ideal law
2.07kgmolair ´ 8.314(kPa ´ m 3 ) ´ 293K
Vair =
(kgmol )( K ) ´ (100kPa)
= 50.3 m3 at 20 oC and 100 kPa

By Assist. Prof. Dr. Wipada Sanongraj 155


1304 211 Chemical Engineering Principles
and Calculations

Another way to solve for Vair at 20 oC and 100 kPa

First…solve for Vair at 20 oC and 94.07 kPa

2.07kgmolair ´ 8.314(kPa ´ m 3 ) ´ 293K


Vair =
( kgmol )( K ) ´ (94.07kPa)

= 53.5 m3 at 20 oC and 94.07 kPa


53.5 m 3 ´ 94.07 kPa
Vair =
100 kPa

= 50.3 m3 at 100 kPa and 20 oC

Example 4.13 : Smokestack Emission and Pollution

A local pollution-solution group has reported the


Simtron co. boiler plant as being an air polluter and
has provided as proof photographs of heavy
smokerstack emissions on 20 different days. As
the chief engineer for the Simtron Co., you know
that your plant is not a source of pollution because
you burn natural gas (essentially methane) and
your boiler plant is operating correctly. Your boss
believes the pollution-solutions group has made an
error in identifying the stack- it must belong to the
company next door that burns coal. Is he correct?
Is the pollution-solutions group correct?

By Assist. Prof. Dr. Wipada Sanongraj 156


1304 211 Chemical Engineering Principles
and Calculations

Solution
Methane contains 2 kg mol of H2 per kgmol of C,
while coal contains 71 kg of C per 5.6 kg of H2 in
100 kg o coal. The coal analysis is equivalent to

1 kgmol C
71 kg C ´ = 5.92 kgmol C
12 kg C

1 kgmol H 2
5.6 kg H 2 ´ = 2.78 kgmol H 2
12 kg H 2

or a ratio of 2.78/5.92 = 0.47 kgmol of H2/kgmol of C.

Suppose that each fuel burns with 40% excess air


and that combustion is completed. We can
compute the mole fraction of water vapor in each
stack gas.

Known Fuel Products

O2 0.21
N2 0.79
1.00

Basis : 1 kg mol C

By Assist. Prof. Dr. Wipada Sanongraj 157


1304 211 Chemical Engineering Principles
and Calculations

Natural Gas
CH 4 + 2O2 ® CO2 + 2 H 2 O

Composition of combustion gases (kgmol)


Components kgmol CO2 H2O Excess O2 N2

C 1.0 1.0
H2 2.0 2.0
Air 0.8 10.5
Total 1.0 2.0 0.80 10.5

Req. O2: 2 Excess O2: 2(0.4) = 0.8 N2: 2.8(79/21) = 10.5

The total kilogram moles of gas produced are 14.3


and the mole fraction H2O is 2/14.3 = 0.14
Coal
1
C + O2 ® CO2 H 2 + O2 ® H 2O
2
Composition of combustion gases (kgmol)
Components kgmol CO2 H2O Excess O2 N2

C 1.0 1.0
H2 0.47 0.47
Air 0.49 6.5
Total 1.0 0.47 0.49 6.5
Req. O2: 1+0.47(1/2) = 1.24 Excess O2: (1.24)(0.4) = 0.49
N2: 1.40(79/21)[1+0.47(1/2)] = 6.5

By Assist. Prof. Dr. Wipada Sanongraj 158


1304 211 Chemical Engineering Principles
and Calculations

The total kilogram moles of gas produced are 8.46


and the mole fraction H2O is 0.47/8.46 = 0.056

If the barometric pressure is, say, 100 kPa, the


stack gas would become saturated and water
vapor would start condense at P*H2O:
Natural gas Coal
Partial pressure 100(0.14) = 14 kPa 100(0.056) = 5.6 kPa
Equivalent temperature 52.5 oC 35 oC

Vapor-Liquid Equilibria for Multicomponent Systems

In this section, we will learn how to calculate the


partial pressures and mole fractions of solutes and
solvents in multicomponent mixtures in which the
gas and liquid phases are at equilibrium.

In a two-phase vapor-liquid mixture at equilibrium, a


component in one phase is in equilibrium with the
same component in the other phase. The equilibrium
relationship depends on the temperature, pressure,
and composition of the mixtures.

By Assist. Prof. Dr. Wipada Sanongraj 159


1304 211 Chemical Engineering Principles
and Calculations

For Ideal Solutions

Henry’s law is primarily used to relate the mole


fraction of one component in the vapor phase to the
mole fraction of the same component in the liquid
phase for a component whose mole fraction
approaches zero, such as a dilute gas dissolved in
as liquid:
Pi = Hixi
where pi is the pressure in the gas phase of the
dilute component at equilibrium at some
temperature, and Hi is the Henry’s law constant.
Pi H i xi
yi = =
Ptot Ptot

Raoult’s law is primarily used for a component whose


mole fraction approaches unity or for solutions of
components quite similar in chemical nature, such as
straight chain hydrocarbons.

Pi = Pi * xi

where P*i is the vapor pressure of component i and xi


is the liquid-phase mole fraction. Note that when xi =
1, pi = p*i
An equilibrium constant, Ki, is defined as follows by
assuming that Dalton’s law applies to the gas phase
(Pi = Ptotyi)
yi Pi*
Ki = =
xi Ptot

By Assist. Prof. Dr. Wipada Sanongraj 160


1304 211 Chemical Engineering Principles
and Calculations

The above equation gives reasonable estimates of


Ki values at low pressure for components well
below their critical temperatures, but yields values
too large for components above their critical
temperatures, at high pressures, and/or for polar
compounds. So Ki was modified to nonideal
mixtures by Sandler (if Tc,i/T > 1.2):

[ 7 .7224 - 7 .534 / Tr , i - 2 .598 ln Tr , i ]


Pc ,i
Ki =
Ptot

Typical problems you may be asked to solve that


involve the use of the equilibrium Ki are:
1) Calculate the bubble point temperature of a liquid
mixture given the total pressure and liquid
composition.
2) Calculate the dew point temperature of a vapor
mixture given the total pressure and vapor
composition.
3) Calculate the related equilibrium vapor-liquid
compositions over the range of mole fractions
from 0 to 1 as a function of temperature given the
total pressure.
4) Calculate the composition of the vapor and liquid
streams, and their respective quantities, when a
liquid of given composition is partially vaporized at
a given temperature and pressure.

By Assist. Prof. Dr. Wipada Sanongraj 161


1304 211 Chemical Engineering Principles
and Calculations

To calculate the bubble point temperature (given the


total pressure and liquid composition), we can write
the following equation as yi = Kixi and we know that
∑yi = 1 in the vapor phase.
n
1= å i =1
K i xi

Where the Ki’s are functions of solely temperature and


n is the number of component. For an ideal solution the
above equation becomes:
n
Ptot = å i =1
Pi * x i

And we might use Antoine’s equation for P*i. Once


the bubble point temperature is determined, the
vapor composition can be calculated from
Pi * x i
yi =
P tot
To calculate the dew point temperature (given the
total pressure and vapor composition, we can write
the following equation as xi = yi/Ki, and we know ∑xi
= 1 in the liquid phase. n
y
1= å i
i =1 K i

in which the K’s are function of temperature as


explained for the bubble point temperature calculation.
n
yi
tot å
For an ideal solution. 1 = P
*
i = 1 Pi

By Assist. Prof. Dr. Wipada Sanongraj 162


1304 211 Chemical Engineering Principles
and Calculations

To calculate the amount of the respective vapor and


liquid phases that evolve at equilibrium when a
liquid of known composition flashes (flash
vaporization) at a known temperature and pressure,
we must use the following equation:

FxFi = Lxi + Vyi

Where F is the moles of liquid to be flashed, L is the


moles of liquid at equilibrium, and V is the moles of
vapor at equilibrium. Introduction of yi = Kixi into the
above equation gives:
æy ö
FxFi = Lçç i ÷÷ + Vyi
è Ki ø

FxFi xFi
yi = =
L L 1
+ (F - L) 1- (1- )
Ki F Ki
where L/F is the fraction of liquid formed on
vaporization. Consequently, since ∑yi = 1 , after
summing the yi’s we want to solve the following
equation:
n
x Fi
1= å L 1
i =1
1- (1 - )
F Ki

By Assist. Prof. Dr. Wipada Sanongraj 163


1304 211 Chemical Engineering Principles
and Calculations

Example 4.14 : Vapor-Liquid Equilibrium Calculation

Suppose that a liquid mixture of 4.0% n-hexane in


n-octane is vaporized. What is the composition of
the first vapor formed if the total pressure is 1.00
atm?

Solution

The mixture can be treated as an ideal mixture


because the components are quite similar. As an
intermediate step, we must calculate the bubble
point temperature.

First we have to calculate the vapor pressures of the


two components:
B
ln( P*) = A -
C +T

A B C
n-hexane (C6) 15.8737 2697.55 -48.784
n-octane (C8) 15.9798 3127.60 -63.633

Basis : 1 kgmol of liquid

By Assist. Prof. Dr. Wipada Sanongraj 164


1304 211 Chemical Engineering Principles
and Calculations

We need to solve the following equation to get the


bubble point temperature:

æ 2697 . 55 ö æ 3127 . 60 ö
760 = exp ç 15 . 8737 - ÷ 0 . 040 + exp ç 15 .9787 - ÷ 0 . 960
è - 48 .784 + T ø è - 63 . 633 + T ø

The solution is T = 393.3 K, where the vapor


pressure of hexane is 3114 mm Hg and the vapor
pressure of octane is 661 mm Hg.

PC*6 3114
yC6 = xC6 = (0.040) = 0.164
Ptot 760

yC8 = 1- 0.164= 0.836

Example 4.15 : Flash Calculation


Calculate the fraction of liquid that will exist at
equilibrium at 150 oF and 50 psia when the liquid
concentrations of the solution to be vaporized are as
follows:
Component Initial liquid K
mole fraction
C2 0.0079 16.20
C3 0.1321 5.2
i-C4 0.0849 2.6
n-C4 0.2690 1.98
i-C5 0.0589 0.91
n-C5 0.1321 0.72
C6 0.3151 0.28
Total 1.0000

By Assist. Prof. Dr. Wipada Sanongraj 165


1304 211 Chemical Engineering Principles
and Calculations

The K values come from the Engineering Data Book


of the Gas Processors Supply Association (1980).
Solution
We want to solve for L/F. Start with an initial guess
of L/F = 1.0.

Stage L/F
1 1.0
2 0.8565
3 0.6567
4 0.5102
5 0.4573
6 0.4511

Gibb’s phase rule is a useful guide in establishing


how many properties, such as pressure and
temperature, have to be specified to definitely fix all
the remaining properties and number of phases that
can coexist for any physical system. The rule can
be applied only to systems in equilibrium. The
Gibbs phase rule states that

F=C–P+2

F = number of degrees of freedom


C = number of components in the system
P = number of phases that can exist in the system

By Assist. Prof. Dr. Wipada Sanongraj 166


1304 211 Chemical Engineering Principles
and Calculations

Phase-rule variables are variables of the kind with


which the phase rule is concerned and they are
intensive properties of the system.
Intensive properties are the ones that do not
depend on the quantity of the material present. The
specific (per unit mass) values are intensive
properties.

Extensive properties are the ones that depend on


how much material we have. The total quantities
are extensive properties.

An example of the use of the phase rule is the ideal


gas law, PV = nRT. In order to be able to determine
the remaining one unknown, you might conclude
that F = 3. However, if we apply the phase rule for
a single phase P = 1 and for a pure gas C = 1, so
that
F=C–P+2=1–1+2=2
How can we explain this apparent paradox with our
previous statement? Since the phase rule is
concerned with intensive properties only, the
following are phase-rule variables in the ideal gas
law:
P, T, and Vˆ

By Assist. Prof. Dr. Wipada Sanongraj 167


1304 211 Chemical Engineering Principles
and Calculations

Thus the ideal gas law would be written as

pVˆ = RT

We can see that by specifying two intensive variables


(F = 2), the third can be calculated.

Example 4.16 Application of the phase rule


Calculate the number of degrees of freedom from
the phase rule for the following materials at
equilibrium:

a)Pure liquid benzene


b) A mixture of ice and water only
c) A mixture of liquid benzene, benzene vapor, and
helium gas
d)A mixture of salt and water designed to achieve
a vapor pressure.

What variables might be specified in each case?

By Assist. Prof. Dr. Wipada Sanongraj 168


1304 211 Chemical Engineering Principles
and Calculations

Solution F=C–P+2
a) C = 1, P = 1, hence F = 1 – 1 + 2 = 2. The
temperature and pressure might be specified in the
range in which benzene remains a liquid.
b) C = 1, P = 2, hence F = 1 – 2 + 2 = 1. Once either
the temperature or the pressure is specified, the
other intensive variables are fixed.
c) C = 2, P = 2, hence F = 2 – 2 + 2 = 2. A pair from
temperature, pressure, or mole fraction can be
specified.
d) C = 2, P = 2, hence F = 2 – 2 + 2 = 2. Since a
particular pressure is to be achieved, you would
adjust the salt concentration and temperature of
the solution.

Partial Saturation and Humidity


Often, the contact time required in a process for
equilibrium (or saturation) to be attained between the
gas and liquid is too long, and the gas is not
completely saturated with the vapor. Then the vapor
is not in equilibrium with a liquid phase, and the
partial pressure of the vapor is less than the vapor
pressure of the liquid at the given temperature. This
condition is called partial saturation.

When the vapor is water vapor and the gas is air, the
special term humidity applies. For other gases or
vapors, the term saturation is used.

By Assist. Prof. Dr. Wipada Sanongraj 169


1304 211 Chemical Engineering Principles
and Calculations

Relative saturation is defined as


Pvapor
RS = = relative saturation
Psatd

where Pvapor = partial pressure of the vapor in the


gas mixture
Psatd = partial pressure of the vapor in the
gas mixture if the gas were saturated
at the given temperature of the
mixture
If the subscript l denotes the vapor,

Pl Pl / Pt Vl / Vt nt massi
RS = = = = =
Pl* Pl* / Pt Vsatd / Vt nsat d masssatd

We can see that relative saturation, in effect,


represents the fractional approach to total saturation.
If you listen to the radio or TV and hear the
announcer say that temperature is 25 oC and the
relative humidity is 60%, he or she implies that

PH 2 O
(100 ) = % R H = 60 %
PH* 2 O

with both the PH2O and the P*H2O being measured at


25 oC.

By Assist. Prof. Dr. Wipada Sanongraj 170


1304 211 Chemical Engineering Principles
and Calculations

Example 4.17 Application of Relative Humidity


The weather report on the radio this morning was that
the temperature this afternoon would reach 94 oF, the
relative humidity would be 43%, the barometer 29.67
in. Hg. partly cloudy to clear, with the wind from SSE at
8 mi/hr. How many pounds of water vapor would be in
1 mi3 of afternoon air? What would be the dew point of
this air?
Solution
The vapor pressure of water at 94 oF is 16.1 in Hg.
We can calculate the partial pressure of the water
vapor in the air from the given percent relative
humidity.

Pw = (1.61 in. Hg)(0.43) = 0.692 in. Hg

(Pair = Pt –Pw = 29.67 – 0.692 = 28.98 in. Hg

Basis : 1 mi3 water vapor at 94 oF and 0.692 in. Hg


3
æ 5280 ft ö 492 o R 0.692 in.Hg 1 lbmol 18 lbH 2O
1 mi ´ ç
3
÷ ´ o
´ ´ 3
´
è 1 mi ø 555 R 29.92 in.Hg 359 ft 1 lbmol
= 1.52 ´108 lb H 2O

Now the dew point is the temperature at which the


water vapor in the air will first condense on cooling at
constant pressure and composition. As the gas is
cooled the relative humidity increases because the
partial pressure of the water vapor is constant while the
vapor pressure of water decreases with temperature.

By Assist. Prof. Dr. Wipada Sanongraj 171


1304 211 Chemical Engineering Principles
and Calculations

When the percent relative humidity reaches 100%

PH 2 O
100 = 100% or PH 2 O = PH* 2 O
PH* 2 O

The water vapor will start to condense. This mean that


at the dew point the vapor pressure of water will be 0.692
in. Hg. The corresponding temperature is 68-69 oF.

Molal Saturation is another way to express vapor


concentration in a gas. It is the ratio of the moles of
vapor to the moles of vapor-free gas.

nvapor
= molal saturation
n vapor - free gas

If subscripts 1 and 2 represent the vapor and the dry


gas, respectively, then for a binary system,
P1+ P2 = Ptot
n1+ n2 = ntot
n1 P1 V1 n1 P1 V1
= = = = =
n2 P2 V2 ntot - n1 Ptot - P1 Vtot - V1

By Assist. Prof. Dr. Wipada Sanongraj 172


1304 211 Chemical Engineering Principles
and Calculations

Humidity refers to the mass of water vapor per


mass of bone-dry air.
(nvapor )(mol.wt .vapor ) mass vapor
H = humidity = =
(ndrygas )(mol.wt .drygas ) mass drygas
Absolute Saturation (humidity) is defined as the
ratio of the moles of vapor per mole of vapor-free
gas to the moles of vapor that would be present per
mole of vapor-free gas if the mixture were
completely saturated at the existing temperature
and total pressure: æ moles vapor ö
ç ÷
ç moles ÷
è vapor - free gas
ø actual
AS =
æ moles vapor ö
ç ÷
ç moles vapor ÷
è - free gas ø saturated

Using the subscripts 1 for vapor and 2 for vapor-


free gas
æ n1 ö æ P1 ö
çç ÷÷ çç ÷÷
è n 2 ø actual è P2 ø actual
% AS = 100 = 100
æ n1 ö æ P1 ö
çç ÷÷ çç ÷÷
è n 2 ø saturated è P2 ø saturated

Since P1 saturated = P*1 and Ptot = P1 + P2

æ P1 ö
çç ÷÷
è total
P - P1 ø P1 æ Ptotal - P1* ö
% AS = 100 = * çç ÷100
æ P1* ö P1 è Ptotal - P1 ÷ø
çç * ÷
÷
è total
P - P1 ø

By Assist. Prof. Dr. Wipada Sanongraj 173


1304 211 Chemical Engineering Principles
and Calculations

From P1/P1* = relative saturation, therefore,

æP - P1* ö
% AS = ( relative saturation ) çç total ÷÷100
è Ptotal - P1 ø

Percent absolute saturation is always less than


relative saturation except at saturated conditions (or
at zero percent saturation) when percent absolute
saturation = percent relative saturation.

Example 4.18 Partial Saturation


The percent absolute humidity of air at 30 oC (86 oF)
and a total pressure of 750 mm Hg (100 kPa) is 20%.
Calculate a) the percent relative humidity, b) the
humidity, c) the partial pressure of the water vapor in
the air. What is the dew point of the air?

Solution
Data from the steam tables are
P*H2O at 86 oF = 1.253 in Hg = 31.8 mm Hg = 4.242 kPa

By Assist. Prof. Dr. Wipada Sanongraj 174


1304 211 Chemical Engineering Principles
and Calculations

To get the relative humidity, PH2O/P*H2O, we need to


find the partial pressure of the water vapor in the air.
This may be obtained from
æ PH 2 O ö
ç ÷ PH 2 O
ç Ptotal - PH O ÷
è ø 750 - PH 2 O
100 ( AH ) = 20 = =
2
100 100
æ PH* 2 O ö 31 . 8
ç ÷
ç Ptotal - PH* O ÷ 750 - 31 . 8
è 2 ø

c) PH2O = 6.58 mm Hg
a) %RH = 100(6.58/31.8) = 20.7%
b) H = [(MWH2O)(nH2O)]/[(MWair)(nair)]

= [18(PH2O)]/[29(Pair)] = 18(6.58)/[(29(750-6.58)] = 0.0055

The dew point is the temperature at which the water


vapor in the air would first commence to condense,
when cooled at constant total pressure, because the
gas becomes completely saturated. This would be
at the vapor pressure of 6.58 mm Hg, or about 5.1
oC.

By Assist. Prof. Dr. Wipada Sanongraj 175

Das könnte Ihnen auch gefallen